Location via proxy:   [ UP ]  
[Report a bug]   [Manage cookies]                
Download as pdf or txt
Download as pdf or txt
You are on page 1of 48

SAS 1

This aims to transform our world and to improve people’s lives and prosperity on a Sustainable
healthy planet. Developmental Goals
This aims to eradicate poverty, hunger, illiteracy and disease. Millennium
Developmental Goals
The following are Pillars of the 2020 National Health Goals, SELECT ALL THAT APPLY: Increase quality and
years of healthy life;
Eliminate health
disparities
Which of the following is one of 4 Provisions of Maternal and Child Health Nursing Provision of nursing care
Practice Throughout the Childbearing‒Childrearing Continuum? of children from birth
through adolescence
A part of a wider 2030 Agenda for Sustainable Development that applies to all countries Sustainable
through partnerships and peace Developmental Goals
The following are Legal Considerations Specific to Maternal-Child Nursing Practice. SELECT A. Informed
ALL THAT APPLY: consent related to fetal
well-being
B. Informed
consent and legal
guardianship for
procedures performed
on children
C. Length of time
between healthcare
incident and child’s
ability to bring lawsuit
D. Identifying and
reporting suspected child
abuse
E. Concepts of
“wrongful birth”,
“wrongful life,” and
“wrongful conception”
Which of the following is the Scope of Nursing Practice that is stated at Philippine Nursing It states that the Nursing
Act of 2002? practice is a holistic
approach, the ideal
functions of being a
nurse, collaborator of
care, provider of health
care education, nurse
educator and finding
more evidence-based
practice by being a nurse
researcher.
The following are Steps of Nursing Process, EXCEPT: Health Restoration
All but one are Statistics related to the Measurement of Maternal and Child Health. Paternal Death Rate
The following are part of Sustainable Developmental Goals, EXCEPT: Improve Maternal Health
SAS 2
Amy Alvarez, 26 years of age, is pregnant with her first child and is experiencing Described genetics in a
significant stress following her recent diagnostic findings. The nurse would be providing way that directly meets
high-quality care if completing which of the following? her learning needs
If it is predicted during a pregnancy that a couple will have a child with Down syndrome, “Would it help if you talk
the couple is asked to make a choice whether they want to continue the pregnancy or to a family who has a
terminate it at that point. To discover how couples feel about having a child with Down child with Down
syndrome, researchers surveyed 2,044 parents on the mailing lists of six nonprofit Down syndrome?”
syndrome organizations. The majority of parents reported they are happy with their
decision to have their child and find their sons and daughters great sources of love and
pride. Ninety-nine percent reported they love their affected son or daughter, 97%
reported being proud of them, 79% felt their outlook on life was more positive because of
them, only 5% felt embarrassed by them, and only 4% regretted having them. In a
following study, siblings also reported their relationship with their affected sibling as a
positive one. Less than 10% felt embarrassed, and less than 5% expressed a desire to
trade their sibling in for another brother or sister (Skotko, Levine, & Goldstein, 2011a,
2011b). Based on the findings of the previous studies, how would the nurse answer Mrs.
Alvarez’s question, “Can you imagine how this will change my life?”
Amy Alvarez’s child is born with Down syndrome. What is a common physical feature of An unusual pattern of
newborns with this disorder that the nurse would want all of the team members to palm creases
recognize?
A woman is aware that she is the carrier of a sex-linked recessive disease (Hemophilia A): There is a 50% chance
her husband is free of the disease. What frequency of this disease could she expect to see her female children will
in her children? inherit the disease
[All the male children will
inherit it]
The nurse is caring for a child with Down syndrome (trisomy 21). This is an example of Chromosome
which type of inheritance? nondisjunction
The nurse prepares a couple to have Karyotype performed. What describes a karyotype? A visual representation of
the chromosome pattern
of an individual
A nurse is interviewing a couple who has come for a preconception visit. The couple asks Dominant
the nurse about inheritance and how it occurs. When describing the concept of genes and
inheritance, the nurse explains that a gene that is expressed when paired with another
gene for the same trait is called:
Down syndrome may occur because of a translocation defect. This means the: Additional chromosome
was inherited because it
was attached to a normal
chromosome.
Both people in a married couple carry the recessive gene for cystic fibrosis. When asked “There is a 1 in 4
about the incidence of any children developing the disorder, what should the nurse chance.”
respond?
When assessing a newborn identified genetically as 46XY21+, What can the nurse expect Poor muscle tone; Palmar
to note on the assessment findings? Select all that apply. crease; Protruding
tongue
SAS 3
Which of the following arteries primarily feeds the anterior wall of the heart? Left anterior descending
artery
When do coronary arteries primarily receive blood flow? During diastole
Which of the following illnesses is the leading cause of death in the US? Coronary artery disease
Which of the following conditions most commonly results in CAD? Atherosclerosis
Atherosclerosis impedes coronary blood flow by which of the following mechanisms? Plaques obstruct the
artery
Which of the following risk factors for coronary artery disease cannot be corrected? Heredity
Exceeding which of the following serum cholesterol levels significantly increases the risk 200 mg/dl
of coronary artery disease?
Which of the following actions is the first priority care for a client exhibiting signs and Enhance myocardial
symptoms of coronary artery disease? oxygenation
Medical treatment of coronary artery disease includes which of the following procedures? Oral medication
administration
Prolonged occlusion of the right coronary artery produces an infarction in which of the Inferior
following areas of the heart?
SAS 4
A 34-year-old female is currently 16 weeks pregnant. You’re collecting the patient’s 34-years-old; Gravida 5,
health history. She has the following health history: gravida 5, para 4, BMI 28, para 4; BMI 28, Family
hypertension, depression, and family history of Type 2 diabetes. Select below all the risk history of Type 2
factors in this scenario that increases this patient’s risk for developing gestational diabetes
diabetes? SELECT ALL THAT APPLY.
When do most patients tend to develop gestational diabetes during pregnancy? usually during the 2-3
trimester of pregnancy
You’re providing an educational class for pregnant women about gestational diabetes. B. “Insulin is a
You discuss the role of insulin in the body. Select all the CORRECT statements about the hormone secreted by the
role and function of insulin: beta cells of the
pancreas.”
C. “Insulin
influences cells by
causing them to uptake
glucose from the blood.”
A 32-year-old female is diagnosed with gestational diabetes. As the nurse you know that 3 hour glucose tolerance
what test below is used to diagnose a patient with this condition? test
A 26-year-old pregnant female is diagnosed with gestational diabetes at 28 weeks “It is important I try to
gestation. You’re educating the patient about this condition. Which statement by the get my fasting blood
patient demonstrates they understood your teaching about gestational diabetes? glucose around 70-95
mg/dL and <140 mg/dL 1
hour after meals.”
Fill-in the blank: When a woman develops gestational diabetes it is during a time in the low; human placental
pregnancy when insulin sensitivity is _____________. This is majorly influenced by lactogen (hPL) and
hormones such as estrogen, progesterone, _______________ and _______________. cortisol
Your patient is 36 weeks pregnant and has gestational diabetes. Which lab result below is Blood glucose 82 mg/dL
euglycemic?
A patient has gestational diabetes and is currently 34 weeks pregnant. Which assessment B. Blood pressure
findings below should you immediately report to the physician? Select all that apply: 190/102
C. Proteinuria
A patient is 35 weeks pregnant. She has gestational diabetes and uncontrolled “It burns when I urinate.”
hyperglycemia. Her current blood glucose is 290 mg/dL. You administer insulin per
physician’s order and recheck the blood glucose level per protocol. It is now 135 mg/dL.
Which statement by the patient requires you to notify the physician?
A baby is born at 37 weeks gestation to a mother with gestational diabetes. As the nurse B. Hypoglycemia
you know at birth that the newborn is at risk for? Select all that apply: C. Respiratory
distress
SAS 5
Nurse Robina has observed her pregnant co-worker arriving to work drunk at least three Report the coworker’s
times in the past month. Which action by Nurse Robina would best ensure client safety behavior to the
and obtain necessary assistance for the co-worker? appropriate supervisor.
Maxima, a 24-year-old pregnant woman is being treated in a chemical dependency unit. Denial
She tells the nurse that she only uses drugs when under stress and therefore does not
have a substance problem. Which defense mechanism is the client using?
Nurse Tara is teaching a Chrisanta Agas about substance abuse. She explains that a Alcohol
genetic component has been implicated in which of the following commonly abused
substances?
Gianna a 22-year old pregnant woman, who is a chronic alcohol abuser is being assessed Cardiovascular
by Nurse Gina. symptoms, such as
Which problems are related to thiamin deficiency? decreases hemoglobin
and hct levels

Gastrointestinal
symptoms, such as
nausea and vomiting
Which medication is commonly used in treatment programs for heroin abusers to Methadone
produce a non-euphoric state and to replace heroin use?
Nurse Christine is teaching a pregnant adolescent women health class about the dangers Sudden death from
of inhalant abuse; the nurse warns about the possibility of: cardiac or respiratory
depression
The newly hired Nurse at Medical Center is assessing a pregnant client who abuses Anxiety, tremors, and
barbiturates and benzodiazepine. The nurse would observe for evidence of which tachycardia
withdrawal symptoms?
The Nurse practicing primary prevention of alcohol abuse would target which groups for Adolescents in their late
educational efforts? teens and young adults in
their early twenties
Lheren is reviewing her lessons in Pharmacology. She is aware that the general Tranquilizing
classification of drugs belonging to the opioid category is analgesic and:
During an initial assessment of a pregnant client admitted to a substance abuse unit for A. How long have you
detoxification and treatment, the nurse asks questions to determine patterns of use of used substances?
substances. Which of the following questions are most appropriate at this time? Select all B. How often do you use
that apply. substances?
C. How do you get
substances into your
body?
E. How much of each
substance do you use?
F. Have you ever felt you
should cut down
substance use?
SAS 6
Which of the following is TRUE in Rh incompatibility? On the first pregnancy of
the Rh(-) mother, the
fetus will not be affected
Which of the following conditions can be triggered by Rh incompatibility between mother Hemolytic disease of the
and fetus? newborn
Which of the following maternal/fetal blood types can lead to hemolytic disease of the Rh negative mother, Rh
newborn? positive fetus
What treatment can prevent the development of sensitization to Rh-D antigen in an Rh Rho(D) immune globulin
negative mother carrying an Rh positive fetus?
You’re educating a group of outpatients about ABO blood typing and compatibility. Which A person with AB+ blood
statement is INCORRECT? can only donate to other
people with either AB+ or
AB- blood.
A 26 year old female is 27 weeks pregnant with her second child. The woman is A-. As the The patient will need to
nurse you know that: receive RhoGAM during
this visit to prevent
hemolytic disease of the
newborn.
The nurse is instructing an unlicensed health care worker on the care of the client with ''Washing my hands and
HIV who also has active genital herpes. Which statement by the health care worker putting on a gown and
indicates effective teaching of standard precautions? gloves is what I must do
before starting care.''
Which statement made to the nurse by a health care worker assigned to care for the ''The other health care
client with HIV indicates a breach of confidentiality and requires further education by the worker and I were out in
nurse? the hallway discussing
how we were concerned
about getting
HIV from our client, so no
one could hear us in the
client's room.''
Which interventions does the home health nurse teach to family members to reduce C. Change the
confusion in the client diagnosed with AIDS dementia? (Select all that apply.) decorations in the home
according to the season.
D. Put the bed close
to the window.
F. Ask the client
when he or she wants to
shower or bathe.
G. Mark off the days
of the calendar, leaving
open the current date.
The home health nurse is making an initial home visit to the client currently living with ''Is there somewhere
family members after being hospitalized with pneumonia and newly diagnosed with AIDS. private in the home we
Which statement by the nurse best acknowledges the client's fear of discovery by his can go and talk?''
family?
SAS 7
A mother asks the nurse if her child’s iron deficiency anemia is related to the child’s Children with iron
frequent infections. The nurse responds based on the understanding of which of the deficiency anemia are
following? more susceptible to
infection than are other
children.
Which statements by the pregnant woman would lead the nurse to suspect that the she A. “I drinks over 3
has iron-deficiency anemia? Select all that apply. cups of milk per day.”
B. “I can’t keep
enough apple juice in the
house; I drink over 10
ounces per day.”
Which of the following foods would the nurse encourage the pregnant mother with iron Potato, peas, and chicken
deficiency anemia?
A pregnant woman is admitted with iron- deficiency anemia and has been receiving iron Reassure the patient this
supplementation. The patient voices concern about how their stool is dark black. As the is a normal side effect of
nurse you would? iron supplementation
A 21-year old client. 6 weeks' pregnant is diagnosed with hyperemesis gravidarum. This Electrolyte Imbalance
excessive vomiting during pregnancy will often result in which of the following
conditions?
A 25 y.o. has arrives to the ER with c/o cramping abdominal pain and mild vaginal Ectopic pregnancy
bleeding. Pelvic exam shows a left adnexal mass that's tender when palpated.
Culdocentesis shows blood in the culdesac. This client probably has which of the following
conditions?
A woman is admitted to the hospital with a ruptured ectopic pregnancy. A laparotomy is Fluid replacement
scheduled.
Preoperatively, which of the following goals is MOST important for the nurse to include
on the client’s plan of care?
The client with hyperemesis gravidarum is at risk for developing: Metabolic acidosis with
dehydration
Which of the following statements best describes hyperemesis gravidarum? Severe nausea and
vomiting leading to
electrolyte, metabolic,
and nutritional
imbalances in the
absence of other medical
problems.
The main reason for an expected increased need for iron in pregnancy is: The mother may have
physiologic anemia due
to the increased need for
red blood cell mass as
well as the fetal requires
about 350-400 mg of iron
to grow
SAS 8
A client makes a routine visit to the prenatal clinic. Although she’s 14 weeks pregnant, the grapelike clusters
size of her uterus approximates that in an 18- to 20-week pregnancy. Dr. Diaz diagnoses
gestational trophoblastic disease and orders ultrasonography. The nurse expects
ultrasonography to reveal:
Of the following terms, which is used to refer to a type of gestational trophoblastic Hydatidiform mole
neoplasm?
Which of the following signs will require a mother to seek immediate medical attention? No fetal movement is felt
on the 6th month
Which of the following signs and symptoms will most likely make the nurse suspect that Passage of clear vesicular
the patient is having hydatidiform mole? mass per vagina
Upon assessment the nurse found the following: fundus at 2 fingerbreadths above the Hydatidiform mole
umbilicus, last menstrual period (LMP) 5 months ago, fetal heart beat (FHB) not
appreciated. Which of the following is the most possible diagnosis of this condition?
Check all that apply to incompetent cervix: A. occurs in 2nd
trimester
B. dilatation is
painless and bloodless
C. shortened
cervical length
D. minimal
symptoms
E. snow-like pattern
on ultrasound
The best time to treat incompetent cervix is between ___ and ____ weeks of pregnancy 12, 14
before the dilatation occurs.
Restriction of activities and cervical cerclage are the treatments for __________ . Incompetent Cervix
A client with incompetent cervix with a previous pregnancy had a cerclage procedure "Go to the hospital to
done at 18 wks in the current pregnancy. The client calls the clinic at 37 wks gestation have the cerclage
because of irregular contractions occurring every five to seven minutes. Which response removed so your cervix
by the nurse is the most appropriate? isn't injured and to allow
the birth to progress"
Which of the following procedure applies sterile tape is threaded in a purse-string manner B
under the submucous layer of the cervix & sutured in place to achieve a closed cervix?

SAS 9
A client in the first trimester of pregnancy arrives at a health care clinic and reports that “I will maintain strict
she has been experiencing vaginal bleeding. A threatened abortion is suspected, and the bedrest throughout the
nurse instructs the client regarding management of care. Which statement, if made by remainder of pregnancy.”
the client, indicates a need for further education?
Bleeding and cramping occur with the cervix closed and membranes intact. threatened
Some of the products are expelled, but the placenta remains attached. Heavy bleeding Incomplete
and cramping doesn’t subside until entire placenta is removed
Any of the 5 spontaneous abortions occurring with 3 consecutive pregnancies. This Habitual
condition is a result of weakened cervix that dilates in the 2nd trimester, and expels the
fetus. This condition is call INCOMPLETE CERVIX.
Embryo or fetus dies but isn’t expelled. It’s often discovered by the physician when no Missed
FHT is present. Fetus must be expelled within 6wks or DIC and/or infections can occur.
All of the products of conception are expelled Complete
During a prenatal screening of a client with diabetes, the nurse should keep in mind that A. Still Birth
the client is at increased risk for which complications? SELECT ALL THAT APPLY B. Gestational
Hypertension
C. Spontaneous Abortion
The following are causes of Spontaneous Abortion, Select All that Apply: A. Abnormal fetal
formation
B. Immunologic
factors: Rh/ABO
incompatibility
C. Implantation
abnormalities
D. Toxoplasmosis
An abortion complicated by infection that occurs in women who have tried to self-abort Septic
or whose pregnancy was aborted illegally using a nonsterile instrument such as a knitting
needle.
The following are types of abortion, Select All that Apply. A. Threatened
B. Incomplete
C. Complete
D. Habitual
E. Early Pregnancy
Loss
SAS 10
A woman, who is 22 weeks pregnant, has a routine ultrasound performed. The ultrasound The patient will need to
shows that the placenta is located at the edge of the cervical opening. As the nurse you have a c-section and
know that which statement is FALSE about this finding: cannot deliver vaginally.
Your patient who is 34 weeks pregnant is diagnosed with total placenta previa. The B. Monitoring vital
patient is A positive. What nursing interventions below will you include in the patient’s signs
care? Select all that apply: E. Placing patient
on side-lying position
F. Monitoring pad
count
G. Monitoring CBC
and clotting levels
A 28 year old female, who is 33 weeks pregnant with her second child, has uncontrolled Preeclampsia
hypertension. What risk factor below found in the patient’s health history places her at
risk for abruptio placentae?
A 36 year old woman, who is 38 weeks pregnant, reports having dark red bleeding. The B. Hard abdomen
patient experienced abruptio placentae with her last pregnancy at 29 weeks. What other C. Fetal distress
signs and symptoms can present with abruptio placentae? Select all that apply: E. Tender uterus
Select all the patients below who are at risk for developing placenta previa: A. A 37 year old woman
who is pregnant with her
7th child.
D. A 20 year old
pregnant female who is a
cocaine user.
You’re performing a head-to-toe assessment on a patient admitted with abruptio Oozing around the IV site
placentae. Which of the following assessment findings would you immediately report to
the physician?
Which statement is TRUE regarding abruptio placenta? Nursing interventions for
this condition includes
measuring the fundal
height
Select all the signs and symptoms associated with placenta previa: A. Painless bright
red bleeding
D. Normal fetal
heart rate
E. Abnormal fetal
position
Disseminated intravascular coagulation (DIC) can occur in __________________. This Abruptio placentae,
happens because when the placenta becomes damaged and detaches from the uterine thromboplastin
wall, large amounts of _____________ are released into mom’s circulation, leading to
clot formation and then clotting factor depletion.
A patient who is 25 weeks pregnant has partial placenta previa. As the nurse you’re “I may start to
educating the patient about the condition and self-care. Which statement by the patient experience dark red
requires you to re-educate the patient? bleeding with pain.”
SAS 11
PROM may occur if the uterus is over-stretched by malpresentation of the fetus, multiple True
pregnancy or excess amniotic fluid.
Cervical incompetence in combination with PROM can be a cause of umbilical cord True
prolapse.
The fetal membranes are so strong that blunt trauma to the abdomen is unlikely to cause False
PROM.
Hypoxia and asphyxia of the woman in labour is a common complication of prolonged False
PROM
A sudden gush of clear watery fluid from the vagina is always seen in cases of PROM. False
Select all the risk factors below that increases a woman’s risk for developing B. Primigravida
preeclampsia: C. BMI 34
D. Pregnant with
twins
E. Maternal history
of preeclampsia
G. History of Lupus
and Diabetes
Your patient is 36 weeks pregnant with severe preeclampsia. The physician has ordered B. Platelets 90,000
lab work to assess for HELLP Syndrome. Which findings on the patient’s lab results μL
correlate with HELLP Syndrome? C. ALT 100 IU/L
D. AST 90 IU/L
F. Abnormal RBC
peripheral smear
Your patient with preeclampsia is started on Magnesium Sulfate. The nurse knows to have Calcium gluconate
what medication on standby?
A 39 week pregnant patient is in labor. The patient has preeclampsia. The patient is Patient reports flushing
receiving IV Magnesium Sulfate. Which finding below indicates Magnesium Sulfate or feeling hot
toxicity and requires you to notify the physician?
In a patient with preeclampsia, what signs and symptoms indicate that the patient has a A. You note bouncing of
high risk of experiencing a seizure due to central nervous system irritability? Select all the foot when it is quickly
that apply: dorsiflexed.
B. Patellar and
bicep deep tendon
reflexes are graded 4+.
SAS 12
A nurse in the labor room is performing a vaginal assessment on a pregnant client in Place the client in
labor. The nurse notes the presence of the umbilical cord protruding from the vagina. Trendelenburg’s position
Which of the following would be the initial nursing action?
When the bag of waters ruptures spontaneously, the nurse should inspect the vaginal Cover the prolapse cord
introitus for possible cord prolapse. If there is part of the cord that has prolapsed into the with sterile gauze wet
vaginal opening the correct nursing intervention is: with sterile NSS and
place the woman on
Trendelenburg position
Which of the following is the nurse’s initial action when umbilical cord prolapse occurs? Place the client in a knee-
chest position in bed
When assessing a laboring client, the nurse finds a prolapsed cord. The nurse should: Elevate the client’s hips
What are the signs that suggest cephalopelvic disproportion? Ruptured bow and lack
of engagement
Which of the following situations is considered a vaginal delivery emergency? Shoulder dystocia
During a vaginal delivery, the Obstetrician declares that a shoulder dystocia has occurred. Flex the woman’s thighs
Which of the following actions by the nurse is appropriate at this time? sharply toward her
abdomen
A type of placental abnormality in which there is unusually deep attachment of the Placenta Accreta
placenta to the uterine myometrium?
A type of placental abnormality that It has 1 or more accessory lobes connected to the Placenta Succenturiata
main placenta that the small lobes may be retained in the uterus leading to hemorrhage?
A type of placental abnormality wherein the cord, instead of entering the placenta Velamentous Insertion of
directly, separates into small vessels that reach the placenta by spreading cross a fold of the Cord
amnion that is usually found with multiple gestation & is associated with anomalies?
SAS 13
A nurse is assigned to care for a client with hypotonic uterine dysfunction and signs of a Oxytocin (Pitocin)
slowing labor. The nurse is reviewing the physician’s orders and would expect to note infusion
which of the following prescribed treatments for this condition?
A nurse in the labor room is preparing to care for a client with hypertonic uterine Provide pain relief
dysfunction. The nurse is told that the client is experiencing uncoordinated contractions measures
that are erratic in their frequency, duration, and intensity. The priority nursing
intervention would be to:
A nurse is developing a plan of care for a client experiencing dystocia and includes several Monitoring fetal heart
nursing interventions in the plan of care. The nurse prioritizes the plan of care and selects rate
which of the following nursing interventions as the highest priority?
A client is admitted to the L & D suite at 36 weeks’ gestation. She has a history of C- Uterine rupture
section and complains of severe abdominal pain that started less than 1 hour earlier.
When the nurse palpates tetanic contractions, the client again complains of severe pain.
After the client vomits, she states that the pain is better and then passes out. Which is the
probable cause of her signs and symptoms?
A pregnant woman who is at term is admitted to the birthing unit in active labor. The Timing and recording
client has only progressed from 2cm to 3 cm in 8 hours. She is diagnosed with hypotonic length of contractions.
dystocia and the physician ordered Oxytocin (Pitocin) to augment her contractions. Which
of the following is the most important aspect of nursing intervention at this time?
The nurse is completing an obstetric history of a woman in labor. Which event in the Uterine fibroid noted at
obstetric history will help the nurse suspects dysfunctional labor in the current time of cesarean
pregnancy? delivery.
The nurse is caring for a primigravid client in the labor and delivery area. Which condition Intrauterine fetal death
would place the client at risk for disseminated intravascular coagulation (DIC)?
The following are common causes of dysfunctional labor. Which of these can a nurse, on Full bladder
her own manage?
In evaluating the effectiveness of IV Pitocin for a client with secondary dystocia. the nurse Progressive cervical
should expect: dilation
During the period of induction of labor, a client should be observed carefully for signs of: Uterine tetany
SAS 14
A woman with a fetus in occipitoposterior position would commonly demonstrate which Intense back pressure
of the following?
What is the most common complication for the mother of an oversized fetus? Uterine dysfunction
Which of the following suggests that the woman has shoulder dystocia? Prolonged second stage
of labor
To aid in fetal rotation in an occipitoposterior position, the nurse should instruct the Assume a hands and
woman to: knees position
This condition predisposes the woman to an oversized fetus: Gestational diabetes
A woman in the second trimester of her pregnancy with a fetus in breech position worries The fetus may still turn
that she would deliver the fetus through cesarean delivery. What should the nurse tell into a cephalic position
her? by week 38 because the
lower extremities fit
more properly in the
fundus.
A pregnant woman with a fetus in face presentation asks if there is any chance that she The baby can be born
would deliver her baby vaginally. The nurse should tell her that: vaginally if the chin is
anterior and the pelvic
diameters are within
normal limits
In brow presentation, the head becomes jammed in the brim of the pelvis as the Obstructed labor
occipitomental diameter presents, and this usually results in:
A baby born in a brow presentation has extreme ecchymosis on the face. The nurse The bruising is normal
should tell the parents that: and would disappear
after several days
In face presentation, when is cesarean birth necessary? If the chin is posterior.
SAS 15
A postpartum nurse is taking the vital signs of a woman who delivered a healthy newborn Increase hydration by
infant 4 hours ago. The nurse notes that the mother’s temperature is 100.2*F. Which of encouraging oral fluids
the following actions would be most appropriate?
The nurse is assessing a client who is 6 hours postpartum after delivering a full-term Instruct the mother to
healthy infant. The client complains to the nurse of feelings of faintness and dizziness. request help when
Which of the following nursing actions would be most appropriate? getting out of bed
A nurse is preparing to perform a fundal assessment on a postpartum client. The initial Ask the mother to
nursing action in performing this assessment is which of the following? urinate and empty her
bladder
The nurse is assessing the lochia on a 1 day postpartum patient. The nurse notes that the Indicates the presence of
lochia is red and has a foul-smelling odor. The nurse determines that this assessment infection
finding is:
When performing a postpartum assessment on a client, the nurse notes the presence of Notify the physician
clots in the lochia. The nurse examines the clots and notes that they are larger than 1 cm.
Which of the following nursing actions is most appropriate?
A nurse in a postpartum unit is instructing a mother regarding lochia and the amount of Eight peripads per day
expected lochia drainage. The nurse instructs the mother that the normal amount of
lochia may vary but should never exceed the need for:
A postpartum nurse is providing instructions to a woman after delivery of a healthy 3 days postpartum
newborn infant. The nurse instructs the mother that she should expect normal bowel
elimination to return:
Select all of the physiological maternal changes that occur during the postpartum period. A. Cervical
involution ceases
immediately
C. Fundus begins to
descend into the pelvis
after 24 hours
A nurse is caring for a postpartum woman who has received epidural anesthesia and is Changes in vital signs
monitoring the woman for the presence of a vulva hematoma. Which of the following
assessment findings would best indicate the presence of a hematoma?
A nurse is developing a plan of care for a postpartum woman with a small vulvar Prepare an ice pack for
hematoma. The nurse includes which specific intervention in the plan during the first 12 application to the area.
hours following the delivery of this client?
SAS 16
A nurse is developing a plan of care for a postpartum woman with a small vulvar Prepare an ice pack for
hematoma. The nurse includes which specific intervention in the plan during the first 12 application to the area.
hours following the delivery of this client?
A new mother received epidural anesthesia during labor and had a forceps delivery after Prepare the client for
pushing 2 hours. At 6 hours PP. her systolic blood pressure has dropped 20 points. her surgery.
diastolic BP has dropped 10 points. and her pulse is 120 beats per minute. The client is
anxious and restless. On further assessment. a vulvar hematoma is verified. After
notifying the health care provider. the nurse immediately plans to:
The postpartum patient who delivered 2 days ago has developed endometritis. Which Cesarean birth
charting entry would the nurse expect to find in this patient’s chart? Select All that Apply: performed secondary to
arrest of dilation
Which of the following statements is/are most accurate about endometritis? B. It is the single
most common
complication after
vaginal delivery
C. It is a
polymicrobial infection
True or False. Endometritis is typically caused by only multiple aerobic organisms. False
The frequency of endometritis is dependent on several factors including, but not limited A. Presence of
to, which of the following? preexisting lower genital
tract infection
B. Type of
anesthesia
C. Length of labor
The usual standard of care for treatment of early-onset endometritis is IV antibiotics, and 24 hours
patients typically are treated until they have been afebrile and asymptomatic for a
minimum of ______ hours.
A systematic review has found that clinicians practicing in low-resource settings can A. Patients whose
provide safe and effective treatment for endometritis with oral and oral-IM regimens, and infection occurred after
that such treatment should be limited to which patients? vaginal delivery
B. Patients who
have evidence of only
mild post caesarean
endometritis
C. A or B
The following are management of the infection of the perineum. EXCEPT: Topical or systemic
antibiotic even before
culture results are
available
The following are conditions that increases the Risk for Post-partal Infection, EXCEPT: Preexisting anemia-
lowers body defense
SAS 17
Which of the following is the primary predisposing factor related to mastitis? Breast injury caused by
overdistention, stasis,
and cracking of the
nipples
A nurse is providing instructions to a mother who has been diagnosed with mastitis. “I need to stop
Which of the following statements if made by the mother indicates a need for further breastfeeding until this
teaching? condition resolves.”
A nurse is preparing a list of self-care instructions for a postpartum client who was B. Wear supportive
diagnosed with mastitis. Select all instructions that would be included on the list. bra
D. Rest during the
acute phase
E. Continue to
breastfeed if the breasts
are not too sore.
Signs and symptoms of mastitis include all of the following except: Diffuse breast tenderness
and warmth
A nurse is preparing a list of self-care instructions for a postpartum client who was A. Wear supportive bra
diagnosed with mastitis. Which of the following instructions would be included on the B. Rest during acute
list? phase
D. Continue to breast-
feed if the breast are not
too sore
A nurse is teaching a postpartum client about breast-feeding. Which of the following The diet should include
instructions should the nurse include? additional fluids
A client who is breast-feeding her newborn infant is experiencing nipple soreness. To Position the infant with
relieve the soreness, the nurse suggests that the client: the ear, shoulder, and hip
in straight alignment with
the infant's stomach
against the mother.
A nurse assigned to care for a postpartum client plans to promote parental-infant bonding Hold and cuddle the
by encouraging the parents to: infant closely
The nurse should instruct a breast-feeding mother that the best way to prevent mastitis is
to do which of the following?
A pregnant woman wants to breastfeed her infant; however, her husband is not Increases the risk that
convinced that there are any scientific reasons to do so. The nurse can give the couple the infant will develop
printed information comparing breastfeeding and bottle-feeding. Which statement is allergies.
most accurate? Bottle-feeding using commercially prepared infant formulas:
SAS 18
You’re developing a plan of care for a patient who is at risk for the development of a deep The nurse will apply
vein thrombosis after surgery. What nursing intervention below would the nurse NOT sequential compression
include in the patient’s plan of care to prevent DVT formation? devices (SCDs) per
physician’s order to the
patient’s lower
extremities every night at
bedtime.
The nurse is assessing a patient, who has many risk factors for the development of a DVT, C. Redness
for signs and symptoms of a deep vein thrombosis. What signs and symptoms below D. Pain
would possibly indicate a deep vein thrombosis is present? E. Warm extremity
F. Swelling
Assessment of Homan’s Sign is the most reliable indicator of a deep vein thrombosis. False
A postpartum client is being treated for DVT. The nurse understands that the client’s Hematuria, ecchymosis,
response to treatment will be evaluated by regularly assessing the client for: and epistaxis
A nurse is caring for a PP client with a diagnosis of DVT who is receiving a continuous Activated partial
intravenous infusion of heparin sodium. Which of the following laboratory results will the thromboplastin time
nurse specifically review to determine if an effective and appropriate dose of the heparin
is being delivered?
While the postpartum client is receiving herapin for thrombophlebitis, which of the Protamine sulfate
following drugs would the nurse Mica expect to administer if the client develops
complications related to heparin therapy?
Deep vein thrombosis occurs in the ______________. Leg
You are at-risk for developing deep vein thrombosis or pulmonary embolism if you: A. Obese
B. Have had recent
surgery
C. Smoking
Pulmonary embolism can occur without deep vein thrombosis. False
Signs and symptoms of deep vein thrombosis (DVT) can include: Redness, warmth,
tenderness and swelling
SAS 19
The nurse observes several interactions between a postpartum woman and her new son. Seldom makes eye
What behavior, if exhibited by this woman, does the nurse identify as a possible contact with her son
maladaptive behavior regarding parent-infant attachment?
Which statement regarding postpartum depression (PPD) is essential for the nurse to be PPD can easily go
aware of when attempting to formulate a plan of care? undetected.
Mothers that have experienced postpartum depression in the past have a decreased risk False
for postpartum depression in the future.
Postpartum depression symptoms are similar to ________ symptoms. Depression unrelated to
childbirth
Symptoms of postpartum depression include: (select all that apply) B. Tiredness
C. Changes in diet
D. Issues with sleep
Why is postpartum depression commonly a missed or under diagnosed problem? Select A. Women do not
All That Apply report symptoms
C. Symptoms are
masked by common
experiences of new
mothers (lack of sleep,
etc)
Risk factors for postpartum depression include:(SELECT ALL THAT APPLY) A. History of abuse
C. History of mental
illness
D. Concurrent life
events
The following are management of Postpartum Depression, EXCEPT: Seldom makes eye
contact with her son
The following are risk factors of Postpartum Depression, EXCEPT: Troubled childhood
[extreme fatigue]
A nurse is caring for a postpartum client suspected of developing postpartum psychosis. C. Symptoms
Which statements accurately characterize this disorder? SELECT ALL THAT APPLY include delusions and
hallucinations
E. The disorder
rarely occurs without
psychiatric history
SAS 20
A woman’s temperature has just risen 0.4°F and will remain elevated during the Progesterone.
remainder of her cycle. She expects to menstruate in about 2 weeks. Which of the
following hormones is responsible for the change?
An infertility specialist is evaluating whether a woman’s cervical mucus contains enough Ferning capacity
estrogen to support sperm motility. Which of the following tests is the physician
conducting?
A client is to receive Pergonal (menotropins) injections for infertility prior to invitro Stimulation of ovulation.
fertilization. Which of the following is the expected action of this medication?
Which test used to diagnose the basis of infertility is done during the luteal or secretory Endometrial biopsy
phase of the menstrual cycle?
A man smokes two packs of cigarettes a day. He wants to know if smoking is contributing "Smoking can reduce the
to the difficulty, he and his wife are having getting pregnant. The nurse's most quality of your sperm”
appropriate response is:
A couple comes in for an infertility workup, having attempted to get pregnant for 2 years. Follicle-stimulating
The woman, 37, has always had irregular menstrual cycles but is otherwise healthy. The hormone (FSH) level
man has fathered two children from a previous marriage and had a vasectomy reversal 2
years ago. The man has had two normal semen analyses, but the sperm seem to be
clumped together. What additional test is needed?
A couple is trying to cope with an infertility problem. They want to know what they can do “Get involved with a
to preserve their emotional equilibrium. The nurse's most appropriate response is: support group. I'll give
you some names."
A woman inquires about herbal alternative methods for improving fertility. Which "You may want to avoid
statement by the nurse is the most appropriate when instructing the client in which licorice root, lavender,
herbal preparations to avoid while trying to conceive? fennel, sage, and thyme
while you are trying to
conceive."
In vitro fertilization-embryo transfer (IVF-ET) is a common approach for women with "IVF is a type of assisted
blocked fallopian tubes or unexplained infertility and for men with very low sperm counts. reproductive therapy
A husband and wife have arrived for their preprocedural interview. The husband asks the that involves collecting
nurse to explain what the procedure entails. The nurse's most appropriate response is: eggs from your wife's
ovaries, fertilizing them
in the lab with your
sperm, and transferring
the embryo to her
uterus."
Nurses should be aware that infertility: Is perceived differently
by women and men.
SAS 21
A nurse in a delivery room is assisting with the delivery of a newborn infant. After the Drying the infant in a
delivery, the nurse prepares to prevent heat loss in the newborn resulting from warm blanket
evaporation by:
A nurse in a newborn nursery receives a phone call to prepare for the admission of a 43- Connect the resuscitation
week-gestation newborn with Apgar scores of 1 and 4. In planning for the admission of bag to the oxygen outlet
this infant, the nurse’s highest priority should be to:
The primary critical observation for Apgar scoring is the: Heart rate
To help limit the development of hyperbilirubinemia in the neonate, the plan of care Monitoring for the
should include: passage of meconium
each shift
When newborns have been on formula for 36-48 hours, they should have a: Screening for PKU
A woman delivers a 3,250 g neonate at 42 weeks’ gestation. Which physical finding is Leathery, cracked, and
expected during an examination if this neonate? wrinkled skin
While assessing a 2-hour old neonate, the nurse observes the neonate to have Do nothing because
acrocyanosis. Which of the following nursing actions should be performed initially? acrocyanosis is normal in
the neonate
The nurse is aware that a neonate of a mother with diabetes is at risk for what Hypoglycemia
complication?
After reviewing the client’s maternal history of magnesium sulfate during labor, which Respiratory depression
condition would the nurse anticipate as a potential problem in the neonate?
Neonates of mothers with diabetes are at risk for which complication following birth? Macrosomia
SAS 22
Which of the following infants is least probable to develop sudden infant death syndrome Baby Gabby who sleeps
(SIDS)? on his back
Baby Chen Ek is a neonate who has a very low-birth-weight. Nurse Cheekie Dhal carefully Bronchopulmonary
monitors inspiratory pressure and oxygen (O2) concentration to prevent which of the dysplasia (BPD)
following?
A nurse in the newborn nursery is monitoring a preterm newborn infant for respiratory Tachypnea and
distress syndrome. retractions
Which assessment signs if noted in the newborn infant would alert the nurse to the
possibility of this syndrome?
A postpartum nurse is providing instructions to the mother of a newborn infant with Continue to breast-feed
hyperbilirubinemia who is being breastfed. The nurse provides which most appropriate every 2-4 hours
instructions to the mother?
A nurse on the newborn nursery floor is caring for a neonate. On assessment the infant is Instillation of the
exhibiting signs of cyanosis, tachypnea, nasal flaring, and grunting. Respiratory distress preparation into the
syndrome is diagnosed, and the physician prescribes surfactant replacement therapy. The lungs through an
nurse would prepare to administer this therapy by: endotracheal tube
A nurse is assessing a newborn infant who was born to a mother who is addicted to drugs. Incessant crying
Which of the following assessment findings would the nurse expect to note during the
assessment of this newborn?
A nurse prepares to administer a vitamin K injection to a newborn infant. The mother asks "Newborn infants are
the nurse why her newborn infant needs the injection. The best response by the nurse deficient in vitamin K,
would be: and this injection
prevents your infant
from abnormal
bleeding."
What are the symptoms of RDS in premature babies? SELECT ALL THAT APPLY A. Breathing
problems at birth that
get worse
B. Blue skin color
(cyanosis)
C. Flaring nostrils
D. Rapid breathing
E. Grunting sounds
with breathing
All of the following are ways to diagnose Respiratory Distress Syndrome in premature A. Baby’s appearance,
babies. SELECT ALL THAT APPLY color, and breathing
efforts
B. Chest X-rays of lungs
C. Blood gas tests
The following are sign and symptoms of infant with a drug dependent mother, EXCEPT: A. Constant movement
possibly leading to -
B. Tremors
C. Frequent sneezing
D. Short palpebral
fissure
SAS 23
Dustin who was diagnosed with Hirschsprung’s disease has a fever and watery explosive Notify the physician
diarrhea. Which of the following would Nurse Cheyenne do first? immediately.
Baby Jonathan was born with cleft lip (CL); Nurse Barbara would be alert that which of the Sucking ability
following will most likely be compromised?
Gianne is being assessed by Nurse Luca-Luca for possible intussusception; which of the Family history
following would be least likely to provide valuable information?
Mr. and Ms. Bane’s child failed to pass meconium within the first 24 hours after birth; this Hirschsprung’s disease
may indicate which of the following?
Which of the following parameters would Nurse Max monitor to evaluate the Vomiting
effectiveness of thickened feedings for an infant with gastroesophageL REFLUX (GER)?
Baby Ellie is diagnosed with gastroesophageal reflux (GER); which of the following nursing Impaired oral mucous
diagnoses would be inappropriate? membrane
Nurse Karen is providing postoperative care for Dustin who has cleft palate (CP); she In the prone position
should position the child in which of the following?
Nurse Joyce is assessing a child’s cultural background, she should keep in mind that: Behavioral patterns are
passed from one
generation to the next
In pediatric gastroesophageal reflux disease (GERD), the immaturity of lower esophageal It has upper and lower
sphincter function is manifested by frequent transient lower esophageal relaxations, sphincters.
which result in retrograde flow of gastric contents into the esophagus. Which statement
about the esophagus is TRUE? Select all that apply.
A nurse review a 3-week old infant’s record and notes the physician documented a Foul smelling, ribbon like
diagnosis of suspected Hirschsprung’s Disease. The nurse knows which symptoms led stool
mom to seek health care?
SAS 24
A baby was born 2 hours ago by Cesarean section. The newborn has a myelomeningocele Infection
with the sac intact and has been placed in an incubator. The nurse, when planning care
for the baby, should focus on potential for:
Appropriate nursing interventions for a newborn's myelomeningocele sac prior to surgery Applying moist saline
include using sterile technique and: dressings
To maintain proper alignment of the hips and lower extremities in a baby with a Hips abducted and feet in
myelomeningocele, the nurse should position the baby with the: a neutral position
Dayan’s child is scheduled for surgery due to myelomeningocele; the primary reason for To reduce the risk of
surgical repair is which of the following? infection
Tiffany is diagnosed with increased intracranial pressure (ICP); which of the following if Increase intrathoracic
stated by her parents would indicate a need for Nurse Charlie to reexplain the purpose for pressure
elevating the head of the bed at a 10 to 20degree angle?
In diagnosing seizure disorder, which of the following is the most beneficial? EEG
After explaining to the parents about their child’s unique psychological needs related to a Cognitive delays
seizure disorder and possible stressors, which of the following interests uttered by them
would indicate further teaching?
Spina bifida is one of the possible neural tube defects that can occur during early Sac formation containing
embryological development. Which of the following definitions most accurately describes meninges and spinal fluid
meningocele?
Janae has a seizure disorder; which of the following would be the lowest priority when Assessing for signs and
caring for her? symptoms of increased
intracranial pressure
(ICP)
Bennett was rushed to the emergency department with possible increased intracranial Diplopia
pressure (ICP); which of the following is an early clinical manifestation of increased ICP in
older children? (Select all that apply.)
SAS 25
You are a daycare provider. One of your children tells you about the spanking she Provide community
received from her mother last night. The girl tells you that her mother got very angry resource
when she "talked-back" to her and this is what usually happens when she is "bad." You recommendations
suspect the child has been maltreated, and following organizational policy, you take her
to the administrator. There are no marks on the child and she says she is not in pain?
Nurse Sol is assessing a parent who abused her child. Which of the following risk factors History of the parent
would the nurse expect to find in this case? having been abused as a
child
A group of nursing students ais currently learning about family violence. Which of the Family violence affects
following is true about the topic mentioned every socioeconomic
level.
During a well-child checkup, a mother tells the Nurse Rio about a recent situation in which Conflictual relationships
her child needed to be disciplined by her husband. The child was slapped in the face for of parents.
not getting her husband breakfast on Saturday, despite being told on Thursday never to
prepare food for him. Nurse Rio analyzes the family system and concludes it is
dysfunctional. All of the following factors contribute to this dysfunction except:
During a home visit to a family of three: a mother, father, and their child, The mother tells The nurse commends the
the community nurse that the father (who is not present) had hit the child on several mother’s efforts and also
occasions when he was drinking. The mother further explains that she has talked her contacts protective
husband into going to Alcoholics Anonymous and asks the nurse not to interfere, so her services.
husband won’t get angry and refuse treatment. Which of the following is the best
response of the nurse?
Which nursing assessment findings are physical signs of sexual abuse of a female child? A. Enuresis
Select all that apply. B. Red and swollen
labia and rectum
C. Vaginal tears
A mother complains to the clinic nurse that her 2 ½-year-old son is not yet toilet trained. Bowel control is usually
She is particularly concerned that, although he reliably uses the potty seat for bowel achieved before bladder
movements, he isn’t able to hold his urine for long periods. Which of the following control, and the average
statements by the nurse is correct? age for completion of
toilet training varies
widely from 24 to 36
months.
The mother of a 14-month-old child reports to the nurse that her child will not fall asleep Give only a bottle of
at night without a bottle of milk in the crib and often wakes during the night asking for water at bedtime.
another. Which of the following instructions by the nurse is correct?
A toddler has recently been diagnosed with cerebral palsy. Which of the following A. Regular
information should the nurse provide to the parents? SELECT ALL THAT APPLY developmental screening
is important to avoid
secondary
developmental delays.
B. Cerebral palsy is
caused by injury to the
upper motor neurons
and results in motor
dysfunction, as well as
possible ocular and
speech difficulties.
D. Parent support
groups are helpful for
sharing strategies and
managing health care
issues.
A mother tells the nurse that she is very worried because her 2-year old child does not do not give snacks to the
finish his meals. What should the nurse advise the mother? child before meals
SAS 26
"The client diagnosed with leukemia has central nervous system involvement. Which “Explain that radiation
instructions should the nurse teach? therapy to the head may
result in permanent hair
loss.”
The nurse analyzes the laboratory values of a child with leukemia who is receiving Use soft small
chemotherapy . The nurse notes that the platelet count is 20,000/ul. Based on the toothbrush for mouth
laboratory result, which intervention will the nurse document in the plan of care? care
A client with acute leukemia is admitted to the oncology unit. Which of the following "Have you had a
would be most important for the nurse to inquire? respiratory infection in
the last 6 months?”
Which statement is correct about the rate of cell growth in relation to chemotherapy? Faster growing cells are
more susceptible to
chemotherapy
A child with cancer has the following lab result: WBC 10,000, RBC 5, and platelet count of Hemorrhage
20,000. When planning this child's care, which risk should the nurse consider most
significant?
A patient with asthma is prescribed to take inhaled Salmeterol and Fluticasone for long- The patient inhales the
term management of asthma. You observe the patient taking these medications. Which Salmeterol first and then
option below best describes the correct order in how to take these medications? waits 5 minutes before
inhaling the Fluticasone.
You're assisting your patient who has asthma to bed. The patient is experiencing a Albuterol
frequent cough and chest tightness. You auscultate the patient's lung fields and note
expiratory wheezes. The patient's peak flow rate is 78% less than their best peak flow
reading. Which medication will provide the patient with the fastest relief from these signs
and symptoms of an asthma attack?
You assist your patient with using their inhaler. The inhaler contains the medication Spacer; thrush
Budesonide. Before administering the inhaler, you will want to connect what device to
the inhaler to help decrease the patient from developing ________?
A patient with asthma is receiving a nebulizer of Cromolyn. The patient reports a burning Reassure the patient this
sensation in the nose along with a horrible taste in their mouth. As the nurse you will? is a temporary side effect
of this medication.
You’re providing discharge teaching to a guardian of a 5-year-old patient who was A. Easily fatigued
admitted with asthma. You discussed the early warning signs of an asthma attack and ask with physical activity
the patient to list some of them. Select all the correct early warning signs verbalized by B. Reduced peak
the patient: flow meter reading
E. Wheezing with
activity
F. Nighttime
coughing
SAS 27
Which of the following instructions would Nurse Courtney include in a teaching plan that Treating streptococcal
focuses on initial prevention for Sheri who is diagnosed with rheumatic fever? throat infections with an
antibiotic
Arrange these parts of the conduction system of the heart in the correct order as an SA Node – Purkinje
action potential would pass through them. Fibers – Atrioventricular
Bundle – R and L bundle
of His – AV Node
Which of these statements regarding the conduction system of the heart is NOT correct? B. The SA node is
Select all that apply. located on the upper wall
of the left atrium.
C. The AV node
conducts action
potentials rapidly
through it.
D. Action potentials
are carried slowly
through the
atrioventricular bundle
A child develops carditis from rheumatic fever. The nurse knows that the areas of the heart muscle and the
heart affected by carditis are the: mitral valve
Most forms of Juvenile Arthritis are autoimmune disorder. TRUE
Juvenile arthritis runs in families and is passed from one generation to the next. FALSE
The overall goal of juvenile arthritis treatment is to control symptoms and stop joint TRUE
damage from happening
A 4 year old is admitted to your unit with a severe case of impetigo. It is important the Contact precautions
nurse follows _______________ while providing care to this patient:
You’re providing education to a group of parents about impetigo. Which statement is “Cases of impetigo most
CORRECT about this disease? likely to occur during the
summer when the
weather is warm.”
A parent brings her child into the clinic due to skin lesions that fail to heal. The lesions are Yellow crusts over the
red, reported to be itchy, and exhibit exudate. You suspect the child may have impetigo. lesions
What is a hallmark finding with this condition?
SAS 28
An adolescent has had a lower leg amputation secondary to a motorcycle accident and is normal and called
complaining of pain in the missing extremity. The nurse should recognize that this is phantom limb sensation
The nurse should assess a teenage child suspected of having early stage scoliosis for A. Curved spinal
which of the following clinical manifestations? Select all that apply: column
B. Unequal
shoulder level
E. Truncal
asymmetry
F. Prominence of
one scapula
The majority of osteosarcomas occur in what age group? Pre-teens and
adolescents
The nurse is evaluating nutritional outcomes for a with anorexia nervosa. Which data best The client gains weight.
indicates that the plan of care is effective?
STIs are most common in which age group? Teens and young adults
up to age 24
Which is the most common STI caused by bacteria? Chlamydia
Gonorrhea is treated with antibiotics. What problem has occurred recently in treatment? The bacteria that cause
gonorrhea have become
resistant to certain
antibiotics
Family dynamics are thought to be a major influence in the development of anorexia The home environment is
nervosa. Which information related to a client's home environment should a nurse overprotective and
associate with the development of this disorder? demands perfection
A 25-year old A client's altered body image is evidenced by claims of "feeling fat" even The client will perceive
though the client is emaciated. Which is the appropriate outcome criterion for this client's an ideal body weight and
problem? shape as normal
Why are behavior modification programs the treatment of choice for clients diagnosed These programs allow
with eating disorders? clients to maintain
control.
SAS 29
A patient is being discharged from the emergency department after being treated for B. Apply ice
epistaxis. In teaching the family first aid measures in the event the epistaxis would recur, compresses to the nose.
what measures should the nurse suggest (select all that apply)? D. Pinch the entire
soft lower portion of the
nose.
E. Partially insert a
small gauze pad into the
bleeding nostril.
A 3-year-old is brought to the ER with coughing and gagging. The parent reports that the Bronchoscopy.
child was eating carrots when she began to gag. Which diagnostic evaluation will be used
to determine if the child has aspirated carrots?
A 5-year-old is brought to the ER with a temperature of 99.5°F (37.5°C), a barky cough, Respiratory treatment of
stridor, and hoarseness. Which nursing intervention should the nurse prepare for? racemic epinephrine.
A 6-week-old is admitted to the hospital with influenza. The child is crying, and the father “The shorter and
tells the nurse that his son is hungry. The nurse explains that the baby is not to have narrower airway of
anything by mouth. The parent does not understand why the child cannot eat. Which is infants increases their
the nurse’s best response to the parent? chances of aspiration so
your child should not
have anything to eat
now.”
A 7-month-old has a low-grade fever, nasal congestion, and a mild cough. What should “I will give my daughter
the nursing care management of this child include? Tylenol every 4 to 6
hours as needed.”
A school-age child has been diagnosed with nasopharyngitis. The parent is concerned “Be sure your child is
because the child has had little or no appetite for the last 24 hours. Which is the nurse’s taking an adequate
best response? amount of fluids. The
appetite should return
soon.”
A parent asks how to care for a child at home who has the diagnosis of viral tonsillitis. “You can give your child
Which is the nurse’s best response? Tylenol every 4 to 6
hours as needed for
pain.”
A school-age child has been diagnosed with strep throat. The parent asks the nurse when “Twenty-four hours after
the child can return to school. Which is the nurse’s best response? the first dose of
antibiotics.”
A school-age child is admitted to the hospital for a tonsillectomy. During the nurse’s post- The child is swallowing
operative assessment, the child’s parent tells the nurse that the child is in pain. Which of excessively.
the following observations would be of most concern to the nurse?
SAS 30
The nurse is admitting a client with suspected tuberculosis (TB) to the acute care unit. The Sputum culture positive
nurse places the client in airborne precautions until a confirmed diagnosis of active TB can for Mycobacterium
be made. Which of the following tests is a priority to confirm the diagnosis? Tuberculosis
The nurse assesses a college-age client complaining of shortness of breath after jogging Asthma
and tightness in his chest. Upon further questioning, the client denies a sore throat, fever,
or productive cough. The nurse notifies the physician that this client’s clinical
manifestations are most likely related to:
Which of the following is a priority to include in the instructions given to a client who has Avoid cigarette smoking
bronchitis?
The nurse is admitting a client who complains of fever, chills, chest pain, and dyspnea. Bronchial
The client has a heart rate of 110, respiratory rate of 28, and a nonproductive hacking
cough. A chest x-ray confirms a diagnosis of left lower lobe pneumonia. Upon auscultation
of the left lower lobe, the nurse documents which of the following breath sounds?
A client with suspected active lung tuberculosis is being scheduled for diagnostic tests. Sputum culture
The nurse anticipates that which diagnostic test will most likely be prescribed to confirm
the diagnosis?
The nurse is instructing a client with moderate persistent asthma on the proper method Beta agonist
for using MDI’s (multidose inhalers). Which medication should be administered FIRST?
The nurse is caring for a client with a pneumothorax. The nurse expects the client to have It will drain air out of the
a chest tube inserted because? thorax, restoring normal
intrathoracic pressure
Nurse Mickey is administering a purified protein derivative (PPD) test to a homeless A positive reaction
client. Which of the following statements concerning PPD testing is true? indicates that the client
has been exposed to the
disease
Nurse Murphy administers albuterol (Proventil), as prescribed to a client with Respiratory rate of 22
emphysema. Which finding indicates that the drug is producing a therapeutic effect? breaths/minute
A 5 year old client is admitted to an acute care facility with influenza. The nurse monitors Pneumonia
the client closely for complications. What is the most common complication of influenza?
SAS 31
Atrial septal defects are characterized by a hole in the interatrial septum that allows FALSE
blood to mix in the right and left atria, which are the lower chambers of the heart.
A patient is diagnosed with a large atrial septal defect. You’re providing information for B. Heart failure
the patient on the complications related to this condition. What topics will you include in C. Stroke
the patient’s education? Select all that apply: D. Pulmonary
Hypertension
You’re caring for a 2-year-old patient who has a large atrial septal defect that needs Left-to-right
repair. This defect is causing complications. These complications are arising from an
abnormal shunting of blood throughout the heart. As the nurse, you know that a
__________________ shunt is occurring in the heart due to the defect.
A 10-year-old has undergone a cardiac catheterization. At the end of the procedure, the Pulses
nurse should first assess:
A heart transplant may be indicated for a child with severe heart failure and: Hypoplastic left heart
syndrome
Bryce is a child diagnosed with coarctation of aorta. While assessing him, Nurse Zach Absent or diminished
would expect to find which of the following? femoral pulses
Appropriate intervention is vital for many children with heart disease in order to go on to Scheduling care to
live active, full lives. Which of the following outlines an effective nursing intervention to provide for
decrease cardiac demands and minimize cardiac workload? uninterrupted rest
periods
The procedure that has to be performed in order to shift the high pressure from the right Jatene Procedure
ventricle to the left ventricle in Transposition of the Great Arteries (TGA) is:
The ductus arteriosus is another fetal structure that is important in the intrauterine life. It Shunts the combined
functions to: cardiac output from the
pulmonary artery to the
systemic circulation
What's an important teaching point for the patient receiving a heart transplant? He'll be at risk for life-
threatening infections
because of the
medications he'll be
taking
SAS 32
A 10-year-old has undergone a cardiac catheterization. At the end of the procedure, the Pulses
nurse should first assess:
A 16-year-old being treated for hypertension has laboratory values of hemoglobin B 16 Beta blockers
g/dL, hematocrit level 43%, sodium 139 mEq/L, potassium 4.4 mEq/L, and total
cholesterol of 220 mg/dL. Which drug does the nurse suspect the patient takes based on
the total cholesterol?
A child born with Down syndrome should be evaluated for which associated cardiac Congenital heart defect
manifestation? (CHD)
A child diagnosed with congestive heart failure (CHF) is receiving maintenance doses of Hypokalemia
digoxin and furosemide. She is rubbing her eyes when she is looking at the lights in the
room, and her HR is 70 beats per minute. The nurse expects which laboratory finding?
A child has been diagnosed with valvular disease following rheumatic fever (RF). During “She will need to take
patient teaching, the nurse discusses the child’s long-term prophylactic therapy with the antibiotics for the
antibiotics for dental procedures, surgery, and childbirth. The parents indicate they rest of her life.”
understand when they say:
A child has been seen by the school nurse for dizziness since the start of the school term. Mitral valve prolapse
It happens when standing in line for recess and homeroom. The child now reports that
she would rather sit and watch her friends play hopscotch because she cannot count out
loud and jump at the same time. When the nurse asks her if her chest ever hurts, she says
yes. Based on this history, the nurse suspects that she has:
A heart transplant may be indicated for a child with severe heart failure and: Hypoplastic left heart
syndrome
Aspirin has been ordered for the child with rheumatic fever (RF) in order to: Reduce joint
inflammation
Congenital heart defects (CHDs) are classified by which of the following? Select all that C. Defects with
apply increased pulmonary
blood flow
D. Defects with
decreased pulmonary
blood flow
E. Mixed defects
F. Obstructive
defects
Family discharge teaching has been effective when the parent of a toddler diagnosed with “I know she will be
Kawasaki disease (KD) states: irritable for 2 months
after her symptoms
started.”
SAS 33
A 13-month-old is discharged following repair of his epispadias. Which statement made If a mucous plug forms in
by the parents indicates they understand the discharge teaching? the urinary drainage
tube, we will irrigate it
gently to prevent a
blockage.”
A child had a urinary tract infection (UTI) 3 months ago and was treated with an oral Urinalysis, urine culture,
antibiotic. A follow-up urinalysis revealed normal results. The child has had no other and VCUG
problems until this visit when the child was diagnosed with another UTI. Which is the
most appropriate plan?
An infant is scheduled for a hypospadias and chordee repair. The parent tells the nurse, “I “The repair is done to
understand why the hypospadias repair is necessary, but do they have to fix the chordee optimize sexual
as well?” Which is the nurse’s best response? functioning when he is
older.”
In addition to increased blood pressure, which findings would most likely be found in a Metabolic acidosis,
child with hydronephrosis? polydipsia, and polyuria
You have a patient that might have a urinary tract infection (UTI). Which statement by the “It burns when I pee.”
patient suggests that a UTI is likely?
Which of the following causes the majority of UTI’s in hospitalized patients? Invasive procedures
A 10-year-old Nik has cystitis; which of the following would Nurse Chinny expect when Dysuria
assessing the child?
Niklaus was born with hypospadias; which of the following should be avoided when a Circumcision
child has such condition?
Which of the following organisms is the most common cause of urinary tract infection Escherichia coli
(UTI) in children?
The following are considered functions of the Urinary System EXCEPT: (Select all that Absorption of digested
apply). molecules
SAS 34
A client is admitted with a diagnosis of hydronephrosis secondary to calculi. The calculi Monitor the client’s
have been removed and post obstructive diuresis is occurring. Which of the following electrolyte levels
interventions should be done?
Your patient with chronic renal failure reports pruritus. Which instruction should you Keep fingernails short
include in this patient’s teaching plan? and clean
A twelve year old student is admitted with dark urine, fever, and flank pain and is Recent sore throat
diagnosed with acute glomerulonephritis. Which would most likely be in this student’s
health history?
Clinical manifestations of acute glomerulonephritis include which of the following? Hematuria and
proteinuria
You expect a patient in the oliguric phase of renal failure to have a 24 hour urine output 400ml
less than:
The most common early sign of kidney disease is: Elevated BUN level
Which finding leads you to suspect acute glomerulonephritis in your 30 year old patient? Hypertension, oliguria,
and fatigue
Polystyrene sulfonate (Kayexalate) is used in renal failure to: Exchange potassium for
sodium
The client enters the outpatient clinic and states to the triage nurse, "I think I have the flu. Systemic lupus
I'm so tired, I have no appetite, and everything hurts." The triage nurse assesses the client erythematosus
and finds a butterfly rash over the bridge of nose and on the cheeks. Which diagnosis
does the nurse expect?
A female client asks the nurse if there are any conditions that can exacerbate systemic Pregnancy is often
lupus erythematosus (SLE). Which is the best nurse response? associated with an SLE
exacerbation.
SAS 35
A patient with tented skin turgor, dry mucous membranes, and decreased urinary output Administering I.V. and
is under nurse Mark’s care. Which nursing intervention should be included the care plan oral fluids
of Mark for his patient?
Shane is admitted in the hospital due to having lower than normal potassium level in her Orange juice and
bloodstream. Her medical history reveals vomiting and diarrhea prior to hospitalization. bananas
Which foods should the nurse instruct the client to increase?
A client with very dry mouth, skin and mucous membranes is diagnosed of having Assessing urinary intake
dehydration. Which intervention should the nurse perform when caring for a client and output
diagnosed with fluid volume deficit?
Which client situation requires the nurse to discuss the importance of avoiding foods high 18-year-old Albert who
in potassium? has renal disease
Camille is diagnosed with hypomagnesemia, which nursing intervention would be Instituting seizure
appropriate? precaution to prevent
injury
Which electrolyte would the nurse identify as the major electrolyte responsible for Sodium
determining the concentration of the extracellular fluid?
Ellerid has a potassium level of 6.5 mEq/L, which medication would nurse Martie Sodium tablets
anticipate?
Which clinical manifestation would lead the nurse to suspect that a client is experiencing Hot, flushed skin and
hypermagnesemia? diaphoresis
Which patient is at more risk for an electrolyte imbalance? An 8 month old with a
fever of 102.3 ‘F and
diarrhea
A patient is admitted to the ER with the following findings: heart rate of 110 (thready Administer hypertonic
upon palpation), 80/62 blood pressure, 25 ml/hr urinary output, and Sodium level of 160. solution of 5% Dextrose
What interventions do you expect the medical doctor to order for this patient? 0.45% Sodium Chloride
and monitor urinary
output
SAS 36
There is a 12 year old patient on the unit with herpes zoster. You would avoid assigning Chicken pox
this patient to a staff member who has never had or been vaccinated for:
A nurse is teaching a client with genital herpes. Education for this client should include an the importance of
explanation of: informing his partners of
the disease
What is an infectious cause of orchitis? Chlamydia
A child appears with a flat pink rash that first appeared on the trunk. Subsequently, the Rubella
rash migrated to the rest of the body. Which of the following is the most likely cause?
Which strain of human papillomavirus (HPV) is associated with the most forms of cancer? 16
Which of the following viruses is responsible for infectious mononucleosus? Epstein Barr virus
Zika virus is primarily transmitted via what route? Mosquito bite
High fever, cough, runny nose, maculopapular rash, and Koplik's spots are seen in which Measles
of the following viral infections?
How is the measles virus primarily spread? Respiratory droplets
Which of the following is an infection of the finger, toe, or nail cuticle with the herpes Herpetic whitlow
simplex virus?
SAS 37
A school nurse is holding a question and answer meeting for parents following a recent Scarlet fever is a viral
outbreak of scarlet fever. Which of the following would the nurse confirm as false? infection
A mother brings her child in for paroxysmal bouts of 10-15 coughs followed by 1-2 deep Pertussis
gasping breaths. It started as a simple cold and cough, which seemed to be getting better
before the gasping paroxysmal coughing started. The coughing is severe enough that he
has vomited several times. What is the most likely diagnosis?
Strawberry tongue and maculopapular ("sandpaper") rash are commonly seen in what Scarlet fever
childhood exanthem?
What organism causes Rocky Mountain spotted fever? Rickettsia rickettsia
Which of the following bacteria is Gram positive cocci that is found in clusters? Staphlococcyous aureus
Which of the following is a fungus that may be responsible for certain cases of diaper Candida albicans
rash?
A 7 year old child contracted scabies with his mother, which is diagnosed the day after ”All family members will
discharge. The client is living at her son’s home, where six other persons are living. During need to be treated.”
her visit to the clinic, she asks a staff nurse, “What should my family do?” The most
accurate response from the nurse is:
When caring for a male client with severe impetigo, the nurse should include which Administering systemic
intervention in the plan of care? antibiotics as prescribed
Nurse Percy discovers scabies when assessing a 10 year old patient who has just been isolate the client’s bed
transferred to the medical-surgical unit from the day surgery unit. To prevent scabies linens until the client is
infection in other patient, the nurse should: no longer infectious
Dr. Desi prescribes an emollient for a client with pruritus of recent onset. The client asks “This prevents
why the emollient should be applied immediately after a bath or shower. How should the evaporation of water
nurse respond? from the hydrated
epidermis.”
SAS 38
Which statement is correct about the rate of cell growth in relation to chemotherapy? Faster growing cells are
more susceptible to
chemotherapy
The client diagnosed with leukemia has central nervous system involvement. Which Explain that radiation
instructions should the nurse teach? therapy to the head may
result in permanent hair
loss.
The nurse analyzes the laboratory values of a child with leukemia who is receiving Use soft small toothbrush
chemotherapy . The nurse notes that the platelet count is 20,000/ul. Based on the for mouth care
laboratory result, which intervention will the nurse document in the plan of care?
The nurse is aware that a major difference between Hodgkin's lymphoma and non- Non-Hodgkin's
Hodgkin's lymphoma is that: lymphoma can manifest
in multiple organs
Which of the following conditions is not a complication of Hodgkin's disease? Myocardial Infarction
Which of the following manifestations would be directly associated with Hodgkin's painless, enlarged lymph
disease? nodes
The Hodgkin’s disease patient described in the question above undergoes a lymph node Reed-Sternberg cells
biopsy for definitive diagnosis. If the diagnosis of Hodgkin’s disease were correct, which of
the following cells would the pathologist expect to find?
A male client is admitted to the hospital with a suspected diagnosis of Hodgkin’s disease. Enlarged lymph nodes
Which assessment findings would the nurse expect to note specifically in the client?
A newly admitted client is diagnosed with Hodgkin’s disease undergoes an excisional Airway
cervical lymph node biopsy under local anesthesia. What does the nurse assess first after
the procedure?
A client admitted with newly diagnosed with Hodgkin’s disease. Which of the following Night sweats
would the nurse expect the client to report?
SAS 39
A 2-month-old infant is brought to the emergency room after experiencing a seizure. The Computed tomography
infant appears lethargic with very irregular respirations and periods of apnea. The parents (CT) scan of the head and
report the baby is no longer interested in feeding and before the seizure, rolled off the dilation of the eyes.
couch. What additional testing should the nurse immediately prepare for?
A child diagnosed with Astrocytoma is having a generalized tonic-clonic seizure. Which Administer blow-by
should the nurse do first? oxygen and call for
additional help.
A child has been diagnosed with a midline brain tumor. In addition to showing signs of Vasopressin.
increased intracranial pressure (ICP), she has been voiding large amounts of very dilute
urine. Which medication does the nurse expect to administer?
The nurse is caring for a 3-year-old with an altered state of consciousness. The nurse Identify her parents and
determines that the child is oriented by asking the child to: state her own name
The mother of a child diagnosed with a potentially life-threatening form of cancer says to This must be a difficult
the nurse, "I don't understand how this could happen to us. We have been so careful to time for you and your
make sure our child is healthy. Which response by the nurse is most appropriate? family. Would you like to
talk about how you are
feeling?"
A preschool-age child undergoing chemotherapy experiences nausea and vomiting. Which Allow the child to choose
of the following would be the best intervention to include in the child's plan of care? what to eat for meals.
Which nursing diagnosis is highest-priority for a child undergoing chemotherapy and Fluid and Electrolyte
experiencing nausea and vomiting? Imbalance
A child with cancer has the following lab result: WBC 10,000, RBC 5, and platelet of Hemorrhage
20,000. When planning this child's care, which risk should the nurse consider most
significant?
Skin reactions are common in radiation therapy. Nursing responsibilities on promoting Washing the area with a
skin integrity should be promoted apart from: mild soap and water and
patting it dry not rubbing
it.
Nausea and vomiting is an expected side effect of chemotherapeutic drug use. Which of Metochlopramide
the following drug should be administered to a client on chemotherapy to prevent nausea
and vomiting?
SAS 40
David, age 15 months, is recovering from surgery to remove Wilms' tumor. Which findings Increased interest in play
best indicates that the child is free from pain?
A child is diagnosed with Wilms' tumor. In planning teaching interventions, what key point Do not put pressure on
should the nurse emphasize to the parents? the abdomen
A child is diagnosed with Wilms' tumor. During assessment, the nurse in charge expects to An abdominal mass
detect:
When assessing a child with Wilm's tumor, the nurse should keep in mind that it is most Palpating the child's
important to avoid which of the following? SELECT ALL THAT APPLY abdomen
The mother of a 4 year old child brings the child to the clinic and tells the pediatric nurse Palpating the abdomen
specialist that the child's abdomen seems to be swollen. During further assessment of the for a mass.
subjective data, the mother tells the nurse that the child has been eating well and that
the activity level of the child is unchanged. The nurse, suspecting the possibility of a
Wilm's tumor, would avoid which of the following during the physical assessment?
A nurse is discussing childhood cancer with the parents of a child in an oncology unit. "The most common site
Which statement by the nurse would be the most accurate? for children's cancer is
the bone marrow."
The nurse should monitor a child with a brain stem glioma receiving vincristine sulfate Constipation
(Oncovin) for which of the following adverse reactions?
While assessing a 2 year-old child with a tentative diagnosis of Wilm’s tumor, the nurse Clothing has become
would be MOST concerned about the mother’s report that? tight around the waist
Which of the following does the nurse assess as a major presenting clinical manifestation Cat’s eye reflex
in the child with a retinoblastoma?
A 12 year old female with cancer is scheduled for radiation therapy. The nurse knows that Fatigue
radiation at any treatment site may cause a certain adverse effect. Therefore, the nurse
should prepare the client to expect:
SAS 41
The liver receives blood from two sources. The _____________ is responsible for pumping hepatic portal vein
blood rich in nutrients to the liver.
Which statements are INCORRECT regarding the anatomy and physiology of the liver? A. The liver has 3 lobes
Select all that apply: and 8 segments
C. The liver turns
urea, a by-product of
protein breakdown, into
ammonia.
You’re providing an in-service on viral hepatitis to a group of healthcare workers. You are Hepatitis A&E
teaching them about the types of viral hepatitis that can turn into chronic infections.
Which types are known to cause ACUTE infections ONLY? Select all that apply:
Which patients below are at risk for developing complications related to a chronic B. An infant who
hepatitis infection, such as cirrhosis and liver cancer? Select all that apply: contracted Hepatitis B at
birth.
C. A 18-year-old female
with Hepatitis C who
reports using IV drugs.
D. A 17-year-old male
with alcoholism and
Hepatitis D.
A patient is diagnosed with Hepatitis A. The parents of the patient asks how a person can Fecal-oral
become infected with this condition. You know the most common route of transmission
is?
Which of the following is NOT a common source of transmission for Hepatitis A? Select all C. Semen
that apply: D. Blood
A 16-year-old patient’s lab work show anti-HAV and IgG present in the blood. As the nurse The patient has
you would interpret this blood work as? recovered from a
previous Hepatitis A
infection and is now
immune to it.
A 10-year-old patient was exposed to the Hepatitis A virus at a local restaurant one week Inform the parent to
ago together with his parents. What education is important to provide to this patient? promptly go to the local
health department to
receive immune globulin.
Select all the ways a person can become infected with Hepatitis B: B. During the birth
process
D. Hemodialysis
E. Sexual intercourse
A 1-year old patient has completed the Hepatitis B vaccine series. What blood result Positive anti-HBs
below would demonstrate the vaccine series was successful at providing immunity to
Hepatitis B?
SAS 42
A nurse is performing a nutritional assessment on a 3-year old patient. Which of the A. Poor wound healing
following clinical findings are suggestive of malnutrition? SELECT ALL THAT APPLY B. Dry hair
C. weak hand grips
D. Impaired coordination
The nurse is teaching a 16-year old who has iron deficiency anemia about foods she Oranges and dark green
should include in her diet. The nurse determines that the patient understands the dietary leafy vegetables
instructions if she selects which of the following from her menu?
A 4-year-old child was born at term, with no congenital anomalies. She is now only 70% of Kwashiorkor
normal body weight. On examination she shows dependent edema of the lower
extremities as well as an enlarged abdomen with palpable fluid wave. Her desquamating
skin shows irregular areas of depigmentation, and hyperpigmentation. Which of the
following nutritional problems is most likely present in this child?
A clinical study is performed involving dietary iron metabolism in adolescents. It is C
observed that intestinal absorption of iron can be enhanced in patients with iron
deficiency anemia by supplementing their diet with another nutrient. Which of the
following vitamins is most likely to have this effect?
A 15-year-old girl has been under a physician's care for the past year after diagnosis of Riboflavin
anorexia nervosa. Her BMI is now 18. On physical examination she has cheilosis.
Laboratory studies show hemoglobin 13.7 g/dL, hematocrit 41.0%, MCV 88 fL, platelet
count 191,055/microliter, and WBC count 4930/microliter. Her serum glucose is 66
mg/dL. Which of the following nutrient deficiencies is most likely to cause her findings?
A study is performed to determine dietary sources of vitamin A. It is observed that some Meat
commonly available foods have more vitamin A than others. Which of the following is
most likely to provide the best source for vitamin A in the diet?
A 5-year-old child develops gradual loss of vision over the past 2 years resulting in A
blindness. On physical examination there is bilateral keratomalacia and corneal scarring.
This child's blindness is most likely to have been prevented by adequate dietary intake of
which of the following vitamins?
An 11-month-old infant is only 60% of ideal body weight. The baby is proportionately Marasmus
small in size. Upon physical examination, the baby is listless and does not respond with
vocalization when touched. A small purplish contusion is noted over the right lower
extremity. Which of the following is the most likely diagnosis?
While at a family reunion, parents are talking about their children. One family unit notes Fluoridation of drinking
that none of their children has had any dental caries, though all of the children have all water
had the same childhood infections and have similar dietary habits. They all brush their
teeth regularly. Children in other family units have had multiple visits to the dentist for
treatment of dental caries. Which of the following is most likely to account for this
observation?
Which of the following is the most abundant mineral in the human body? Calcium
SAS 43
Which of the following situations increase risk of lead poisoning in children? playing in the park with
heavy traffic and with
many vehicles passing by
The nurse answers a call bell and finds a frightened mother whose child, the patient, is The nurse should clear
having a seizure. Which of these actions should the nurse take? the area and position the
client safely.
A 5-year-old is admitted to the hospital with a subarachnoid hemorrhage has complaints Intracranial pressure
of severe headache, nuchal rigidity, and projectile vomiting. The nurse knows lumbar (ICP) is increased
puncture (LP) would be contraindicated in this client in which of the following
circumstances?
Select the main structures below that play a role with altering intracranial pressure: A. Brain
C. Cerebrospinal Fluid
D. Blood
A patient is being treated for increased intracranial pressure. Which activities below A. Coughing
should the patient avoid performing? B. Sneezing
D. Valsalva maneuver
E. Vomiting
Which of the following is contraindicated in a patient with increased ICP? Lumbar puncture
You’re collecting vital signs on a patient with ICP. The patient has a Glasgow coma Scale Rectal
rating of 4. How will you assess the patient’s temperature?
During the assessment of a patient with increased ICP, you note that the patient’s arms Decerebrate posturing
are extended straight out and toes pointed downward. You will document this as:
While positioning a patient in bed with increased ICP, it important to avoid? Flexion of the hips
During the eye assessment of a patient with increased ICP, you need to assess the The eyes will be in a fixed
oculocephalic reflex. If the patient has brain stem damage what response will you find? position as the head is
moved side to side.
SAS 44
The nurse is providing education to the parents of a 2-year-old boy with hydrocephalus "watch for changes in his
who has just had a ventriculoperitoneal shunt placed. Which information is most behavior or eating
important for the parents to be taught? patterns"
The nurse is examining a 15-month-old child who was able to walk at the last visit and Schedule a full evaluation
now can no longer walk. What would be the nurse's best intervention in this case? since this may indicate a
neurologic disorder.
Which of the following terms describes the congenital abnormality of the forebrain in Microcephaly
which an infant is born with a diminished brain size?
Anencephaly and other neural tube defects have been linked to maternal deficiency of Folate
what nutrient?
Spina bifida is one of the possible neural tube defects that can occur during early Sac formation containing
embryological development. Which of the following definitions most accurately describes meninges and spinal fluid
meningocele?
All of the following are types of Spina Bifida, EXCEPT: Hemophilia
Are there any treatment for Spina Bifida? Yes
Which of the following are health issues a person with Spina Bifida might have? Skin conditions
Can a person with Spina Bifida fully participate in life? Yes
Do people with Spina Bifida have mental issues? Sometimes
SAS 45
What are the symptoms of cerebral palsy? A. Baby doesn’t roll over
in either direction?
B. Baby cannot bring her
hands together
C. Baby has difficulty
bringing her hands to her
mouth
What causes cerebral palsy? A. Brain damage that
happened before, during
or immediate after birth
B. An infection such as
meningitis
C. Head Injury
D. The cause is unknown
Cerebral Palsy is usually diagnosed during the first or second year after birth. True
A 7-month-old child has been diagnosed with cerebral palsy (CP). Which of the following Positive grasp reflex
signs/ symptoms would the nurse assess as consistent with the diagnosis?
The nurse is planning care for a ten month-old infant with bacterial meningitis. Which of Measure head
the following nursing measures would be appropriate for the nurse to do? circumference
You’re educating a patient about treatment options for Guillain-Barré Syndrome. Which “Immunoglobin
statement by the patient requires you to re-educate the patient about treatment? therapies are treatment
options available for this
syndrome but are most
effective when given
within 4 weeks of the
onset of symptoms.”
Which tests below can be ordered to help the physician diagnose Guillain-Barré C. Lumbar puncture
Syndrome? Select all that apply: D. Electromyography
E. Nerve Conduction
Studies

You’re teaching a group of nursing students about Guillain-Barré Syndrome and how it A. Altered body
can affect the autonomic nervous system. Which signs and symptoms verbalized by the temperature regulation
students demonstrate they understood the autonomic involvement of this syndrome? C. Cardiac dysrhythmias
Select all that apply: D. Orthostatic
hypotension
E. Bladder distension
You’re about to send a patient for a lumbar puncture to help rule out Guillain-Barré Void
Syndrome. Before sending the patient you will have the patient?
Your patient is back from having a lumbar puncture. Select all the correct nursing B. Keep the patient flat.
interventions for this patient? Select all that apply: D. Encourage the patient
to consume liquids
regularly.
SAS 46
You’re educating a 15-year-old female about possible triggers for seizures. Which “I will limit my alcohol
statement requires you to reeducate the patient about the triggers? intake to 2 glasses of
wine per day.”
A patient who is experiencing a tonic-clonic seizure is experiencing a focal (partial) False
seizure.
A 7-year-old male patient is being evaluated for seizures. While in the child’s room talking Absence
with the child’s parents, you notice that the child appears to be daydreaming. You time
this event to be 10 seconds. After 10 seconds, the child appropriately responds and
doesn’t recall the event. This is known as what type of seizure?
Your patient has a history of epilepsy. While helping the patient to the restroom, the Lay the patient down on
patient reports having this feeling of déjà vu and seeing spots in their visual field. Your their side with a pillow
next nursing action is to? underneath the head.
Keeping the previous question in mind, the patient is now experiencing characteristics of Initiate the emergency
a tonic-clonic seizure. The seizure started at 1402 and it is now 1408, and the patient is response system
still experiencing a seizure. The nurse should?
Your patient has entered the post ictus stage for seizures. The patient’s seizure presented Sleepy, headache, and
with an aura followed by body stiffening and then recurrent jerking. The patient had soreness
incontinence and bleeding in the mouth from injury to the tongue. What is an expected
finding in this stage based on the type of seizure this patient experienced?
You’re developing discharge instructions to the parents of a child who experiences atonic “Be sure your child wears
seizures. What information below is important to include in the teaching? a helmet daily.”
You’re assessing a patient who recently experienced a focal type seizure (partial seizure). “My friend reported that
As the nurse, you know that which statement by the patient indicates the patient may during the seizure I was
have experienced a focal impaired awareness (complex partial) seizure? staring off and rubbing
my hands together, but I
don’t remember doing
this.”
You have a patient who has a brain tumor and is at risk for seizures. In the patient’s plan A. Oxygen and suction at
of care you incorporate seizure precautions. Select below all the proper steps to take in bedside
initiating seizure precautions: E. Padded bed rails
F. Remove restrictive
objects or clothing from
patient’s body
G. IV access
You’re patient is scheduled for an EEG (electroencephalogram). As the nurse you will: B. Hold seizure
medications until after
the test.
D. Wash the patient’s
hair prior to the test.
SAS 47
A client has suffered a spinal cord injury after a fall. When he is brought in for care, the Raise the head of the
client experiences diaphoresis and headache. The nurse notes that his blood pressure is bed, lower the legs, and
174/102 mmHg. Which action should the nurse perform first? remove constrictive
clothing
An 18-year-old client has suffered a spinal cord injury in which he is experiencing spinal Ask the client if he would
shock and cannot feel his legs. Twenty-four hours after the injury, the client tells the like to speak with a
nurse, “this will be good for me. I can handle this and I’m doing fine.” Which response spiritual advisor or social
from the nurse is most likely indicated? worker
A client with a spinal cord injury has difficulty determining when the bladder needs The area over the
emptied. The nurse teaches the client about tapping to stimulate voiding. How would the bladder is tapped to
nurse describe tapping to this client? stimulate the bladder
muscles
The most appropriate prevention against spinal cord injury in a 17-year-old patient. Use seat belts in the car
Which of the following actions is correct?
The nurse knows that a client suffering from complete paralysis from the waist down Paraplegia
would have the condition of which of the following?
The nurse is on a rapid response team that has responded to a code blue. The 15-year-old Jaw thrust maneuver
victim is a visitor who has fallen down a flight of stairs and is unresponsive. A cervical
spine injury is suspected. In order to perform CPR safely, what is the best initial
intervention?
An 18-year-old patient with her parents has suffered a spinal cord injury after a driving Flaccid paralysis and
accident. When first brought to the emergency room, the patient is suffering from spinal anesthesia in the lower
shock. Which symptoms best describe this condition? extremities
The client has a spinal cord injury and is suffering from spinal shock. Which of the Bradycardia
following is an expected symptom in spinal shock?
A client is being seen in the emergency department after a spinal cord injury. The client The point at which the
initially states an inability to feel anything below the shoulders. At which point would this client has a Glasgow
client need to be intubated with an endotracheal tube? Coma Score of 8
The nursing care plan of a client with spinal cord injury should include which of the A. Maintain vital signs
following? Select all that apply. B. Provide skeletal
traction
D. Prepare client for
surgical stabilization of
spine
E. Administer
antispasmodics
SAS 48
When using a Snellen alphabet chart, the nurse records the client’s vision as 20/40. Which The client can see at 20
of the following statements best describes 20/40 vision? feet what the person
with normal vision can
see at 40 feet.
The clinic nurse is preparing to test the visual acuity of a client using a Snellen chart. The right eye is tested
Which of the following identifies the accurate procedure for this visual acuity test? followed by the left eye,
and then both eyes are
tested.
The clinic nurse notes that the following several eye examinations, the physician has 20/200 vision
documented a diagnosis of legal blindness in the client’s chart. The nurse reviews the
results of the Snellen’s chart test expecting to note which of the following?
The client’s vision is tested with a Snellen’s chart. The results of the tests are documented The client can read only
as 20/60. The nurse interprets this as: at a distance of 20 feet
what a client with normal
vision can read at 60
feet.
How do tears benefit the eyes? A. By moistening them
B. By coating them with
antibodies
C. By delivering nutrients
to them
What does 20/20 vision mean? Can see objects clearly at
20 feet
Which of these will help reduce eyestrain when you work at a computer? A. Frequent blinking
B. Shifting focus from
near to far object
C. Eliminating glare on
the screen
Which of these vision problems does refractive surgery correct? A. Nearsightedness
B. Farsightedness
C. Astigmatism
Which of these diseases is a common cause of blindness? Diabetes
Which of these eye care professionals is qualified to do surgery? Opthalmologist
SAS 49
The best way to differentiate between a Chalazion and a Hordeolum is: A. The presence of pus
C. Pain
Which of the following is not effective in the treatment of chalazion? Systemic antibiotic
A patient presents with a nontender, painless, nodule involving a meibomian gland. Chalazion
Which of the following is the most likely diagnosis?
The underlying pathology causing blepharitis is: A dysfunction in the
meibomian glands
Which of the following organisms is not a common pathogen causing bacterial Chlamydia trachomatis
conjunctivitis?
A 17-year-old girl comes to your office with a 1-day history of red eye. She describes not Bacterial Conjunctivitis
being able to open her right eye in the morning because of crusting and discharge. The
right eye feels swollen and uncomfortable, although there is no pain. On examination, she
has a significant redness and injection of the right bulbar and palpebral conjunctivae.
There is a mucopurulent discharge present. No other abnormalities are present on
physical examination. Her visual acuity is normal.
A patient is evaluated in the office with a red eye. The patient awoke with redness and a Viral Conjunctivitis
watery discharge from the eye. The eyelids were not matted together. Examination
reveals a palpable preauricular node. Which of the following is the most likely diagnosis?
A 7 month old female is brought to your clinic by his mother who reports the child has Dacrocystitis
had swelling of the nasal corner of the left eye. If left untreated what condition can
develop?
An important nursing intervention in the care of a child with conjunctivitis is: applying intermittent
warm, moist compresses
to remove crusts in the
eye area
Which of the following instructions by the nurse is most appropriate for a client using Discard all opened or
contact lenses who is diagnosed with bacterial conjunctivitis? used lens care products
SAS 50
A 15-year-old patient presents with unilateral hearing loss. Weber reveals lateralization to Impacted cerumen in the
the right ear. Rinne test reveals the following: RIGHT: bone conduction = 10 seconds, air right ear
conduction = 5 seconds; LEFT: bone conduction = 5 seconds, air conduction = 10 seconds.
Which of these other physical exam findings is to be expected?
A nurse is caring for a toddler who has acute otitis media. Which of the following is the Administer analgesics
priority action for the nurse to take?
The part of the ear that contains the receptors for hearing is the: Cochlea
The ear bones that transmit vibrations to the oval window of the cochlea are found in the: Middle ear
Nerve deafness would most likely result from an injury or infection that damaged the: Cochlear nerve
Physiologically, the middle ear, containing the three ossicles, serves primarily to: Amplify the energy of
sound waves entering
the ear
There is a considerable debate about the use of tympanostomy tubes in the management Improve hearing
of recurrent otitis media in children. Tympanostomy tube placement has been proven to:
Which of the following is the most likely organism in a 2 year-old child with acute otitis Streptococcus
media? pneumoniae
This usually occurs following a respiratory tract infection? Acute Otitis Media
It is inflammation of the external ear canal: External Otitis Media
SAS 51
Jaz is a nine (9)-year-old child admitted to a psychiatric treatment unit accompanied by Interview Jaz with his
Mr. and Mrs. Chenes. To establish trust and position of neutrality, which action would the parents together,
nurse take? observing their
interaction
Nurse Rain is a nurse practicing primary prevention for psychiatric disorders in children. Family history of mental
On which of the following risk factors would he focus? illness
Nurse Kaya, a school nurse, is meeting with the school and health treatment team about a Increased ability to
child who has been receiving methylphenidate (Ritalin) for two (2) months. The meeting is concentrate on tasks
to evaluate the results of the child’s medication use. Which behavior change noted by the
teacher will help determine the medication’s effectiveness.
Which behavioral assessment in a child is most consistent with a diagnosis of conduct Physical aggression
disorder? toward others
The parents of Mika, a child with attention deficit hyperactivity disorder, tell the nurse Actively listen to the
they have tried everything to calm their child and nothing has worked. Which action by parents’ concern before
the nurse is most appropriate initially? planning interventions.
Nurse Gloria questions the parents of a child with oppositional defiant disorder about the Generational boundaries
roles of each parent in setting rules of behavior. The purpose for this type of questioning
is to assess which element of the family system?
Nurse Ting-ting reinforces the behavioral contract for a child having difficulty controlling It will assist the child to
aggressive behaviors on the psychiatric unit. Which of the following is the best rationale develop more adaptive
for this method of treatment? coping methods.
The school nurse assesses Brook, a child newly diagnosed with attention deficit A. Constant fidgeting and
hyperactivity disorder (ADHD). Which of the following symptoms are characteristic of the squirming
disorder? Select all that apply. C. Difficulty paying
attention to details
D. Easily distracted
F. Talking
constantly, even when
inappropriate
Ritalin is the drug of choice for children with ADHD. The side effects of the following may Increased attention span
be noted: and concentration
The nurse is developing a care plan for a teenage patient with attention deficit Assign a staff member
hyperactivity disorder who is at high risk for self-harm due to poor judgment, high risk- one-to-one close
taking behaviors, and impulsivity. Which of the following is the priority nursing observation until the
intervention? treatment team
determines she is no
longer a risk for self-
harm.
SAS 52
A client is experiencing a severe panic attack. Which nursing intervention would meet this Stay with the client and
client's immediate need? offer reassurance of
safety
A nurse has been caring for a client diagnosed with post-traumatic stress disorder. What The client will not require
short-term, realistic, correctly written outcome should be included in this client's plan of medication to obtain
care? adequate sleep by
discharge
Which nursing diagnosis would best describe the problems evidenced by the following Post-trauma syndrome
client symptoms: avoidance, poor concentration, nightmares, hypervigilance, exaggerated
startle response, detachment, emotional numbing, and flashbacks?
A client presents at the urgent care clinic and states, "My heart feels like it's skipping A. Strenuous exercise
beats." The client also reports always feeling cold, and has a BMI of 18. The nurse C. Extreme perfectionism
suspects anorexia. Which other clinical manifestation should the nurse assess? (Select all D. Obsession over body
that apply.) shape
E. Rigidity and the need
to control situations
The parents of a teenage girl bring their daughter to the healthcare provider, citing their Moderate
increasing concern about the teen's weight and their suspicion that their daughter has
anorexia nervosa (AN). During assessment, the nurse notes a BMI of 16.75 kg/m2. In
which category does the client fall, according to DSM-5 criteria and considering the
severity of anorexia nervosa?
The client with anorexia nervosa is improving if: Weight gain
The characteristic manifestation that will differentiate bulimia nervosa from anorexia have episodic binge
nervosa is that bulimic individuals eating and purging
A nursing diagnosis for bulimia nervosa is powerlessness related to feeling not in control Patient will learn
of eating habits. The goal for this problem is: problem solving skills
In the management of bulimic patients, the following nursing interventions will promote a Discuss their eating
therapeutic relationship EXCEPT: behavior
A nurse at Sky Castle Medical Center is developing a care plan for a female client with Encourage the client to
post-traumatic stress disorder. Which of the following would she do initially? verbalize thoughts and
feelings about the
trauma.
SAS 53
A client is diagnosed with major depressive disorder. Which nursing diagnosis should a Social isolation related to
nurse assign to this client to address a behavioral symptom of this disorder? poor self-esteem as
evidenced by secluding
self in room
A nurse assesses a client suspected of having major depressive disorder. Which client The client has maxed-out
symptom would eliminate this diagnosis? charge cards and
exhibits promiscuous
behaviors.
A nurse reviews the laboratory data of a client suspected of having major depressive Thyroid-stimulating
disorder. Which laboratory value would potentially rule out this diagnosis? hormone (TSH) level of
6.2 U/Ml
What is the rationale for a nurse to perform a full physical health assessment on a client Depression can generate
admitted with a diagnosis of major depressive disorder? somatic symptoms that
can mask actual physical
disorders.
Which statement about an individual with a personality disorder is true? The individual typically
remains in the
mainstream of society,
although he has
problems in social and
occupational roles.
Chai is with an anxious, fearful personality who has difficulty accomplishing work Ineffective coping
assignments because of his fear of failure. She has been referred to the employee
assistance program because of repeated absences from work and evidence of an alcohol
problem. Which nursing diagnosis would be most appropriate?
Another term that has been previously used for bipolar disorder is___________________. Manic depression
Bipolar disorder is what type of disorder? Mood
Medically speaking, mania is defined as... Abnormally elevated
moods
Bipolar Disorder is hereditary. False
SAS 54
A nurse is monitoring a new mother in the PP period for signs of hemorrhage. Which of An increase in the pulse
the following signs, if noted in the mother, would be an early sign of excessive blood loss? from 88 to 102 BPM
Methergine or Pitocin is prescribed for a woman to treat PP hemorrhage. Before Blood pressure
administration of these medications, the priority nursing assessment is to check the:
Methergine or Pitocin are prescribed for a client with PP hemorrhage. Before Peripheral vascular
administering the medication(s), the nurse contacts the health provider who prescribed disease
the medication(s) in which of the following conditions is documented in the client’s
medical history?
Which of the following complications is most likely responsible for a delayed postpartum Uterine subinvolution
hemorrhage?
A client is diagnosed with gestational hypertension and is receiving magnesium sulfate. Deep tendons reflexes 2+
Which finding would the nurse interpret as indicating a therapeutic level of medication?
A woman hospitalized with severe preeclampsia is being treated with hydralazine to Tachycardia
control blood pressure. Which of the following would the lead the nurse to suspect that
the client is having an adverse effect associated with this drug?
After reviewing a client's history, which factor would the nurse identify as placing her at Mother had gestational
risk for gestational hypertension? hypertension during
pregnancy
Which of the following would the nurse have readily available for a client who is receiving Calcium gluconate
magnesium sulfate to treat severe preeclampsia?
The nurse is assessing a pregnant woman with gestational hypertension. Which of the Hyperreflexia
following would lead the nurse to suspect that the client has developed severe
preeclampsia?
The perinatal nurse is caring for a woman in the immediate postpartum period. Uterine atony
Assessment reveals that the woman is experiencing profuse bleeding. The most likely
etiology for the bleeding is:
SAS 55
What is the correct depth of chest compressions in an adult? Up to 2 inches

[Between 2 and 2.4


inches]
A victim probably has a neck injury. What is the correct way to open the airway? Jaw thrust
How long should a pulse check last? No more than 10 seconds
Where should you check for a pulse in an adult? Carotid artery
Where should you check for a pulse in an infant? Brachial artery
A child is gasping for breath but has a pulse rate of 100 per minute. The rescuers should: Give 1 breath every 3 to
5 seconds
A child is not breathing but has a pulse rate of 50 per minute. The rescuers should: Start CPR beginning with
compressions
A 18-year-old girl who has been eating steak in a restaurant abruptly stands up and grabs Use abdominal thrusts
her neck. The rescuer determines that the victim is choking. The best response is to:
An infant who had been choking becomes unresponsive. The rescuer should: Begin CPR
Efforts to relieve choking should be stopped when: A. The obstruction is
removed
B. The victim becomes
unresponsive
C. The victim begins
breathing normally
SAS 56
You are caring for a patient who has been taking methylergonovine maleate (Methergine) Dilatation and curettage
for postpartum hemorrhage. It is not helping to control the hemorrhage. What procedure
will most likely be ordered for this patient?
The client has undergone a dilation and curettage (D & C) following a spontaneous B. Use sanitary pads until
abortion at 8 weeks. To promote an optimal recovery, what information should the nurse vaginal bleeding has
include in the discharge teaching? (Select all that apply.) stopped
C. Strenuous sport
activities should be
postponed until bleeding
stops
E. Referral for grief
counseling
A nurse is performing an assessment of a client who is scheduled for a cesarean delivery. Fetal heart rate of 180
Which assessment finding would indicate a need to contact the physician? beats per minute
A client in labor is transported to the delivery room and is prepared for a cesarean Supine position with a
delivery. The client is transferred to the delivery room table, and the nurse places the wedge under the right
client in the: hip
Nurse Cecilia is caring for a client who has undergone a vaginal hysterectomy. The nurse Elevating the knee gatch
avoids which of the following in the care of this client? on the bed
After a hysterectomy, clients may feel incomplete as women. The statement that should I feel washed out; there
alert nurse Gina to this feeling would be: isn’t much left
When planning care with a client during the postoperative recovery period following an Surgical menopause will
abdominal hysterectomy and bilateral salpingo-oophorectomy, nurse Frida should include occur
the explanation that:
A client is admitted to the L & D suite at 36 weeks’ gestation. She has a history of C- Uterine rupture
section and complains of severe abdominal pain that started less than 1 hour earlier.
When the nurse palpates tetanic contractions, the client again complains of severe pain.
After the client vomits, she states that the pain is better and then passes out. Which is the
probable cause of her signs and symptoms?
The nurse should realize that the most common and potentially harmful maternal Hypotension
complication of epidural anesthesia would be:
Which measure would be least effective in preventing postpartum hemorrhage? Massage the fundus
every hour for the first 24
hours following birth
SAS 57
The nurse is giving dietary instructions on a client who is on a vegan diet. The nurse Vitamin D.
provides dietary teaching focus on foods high in which vitamin that may be lacking in a
vegan diet?
The nurse is teaching a client who has iron deficiency anemia about foods she should Oranges and dark green
include in her diet. The nurse determines that the client understands the dietary leafy vegetables.
instructions if she selects which of the following from her menu?
When planning a diet with a pregnant woman, the nurse's FIRST action would be to: Review the woman's
current dietary intake.
A pregnant woman with a body mass index (BMI) of 22 asks the nurse how she should be 2 to 5 lbs during the first
gaining weight during pregnancy. The nurse's BEST response would be to tell the woman trimester, then a pound
that her pattern of weight gain should be approximately: each week until the end
of pregnancy.
A pregnant woman at 7 weeks of gestation complains to her nurse midwife about Eat a high-protein snack
frequent episodes of nausea during the day with occasional vomiting. She asks what she before going to bed
can do to feel better. The nurse midwife could suggest that the woman:
A pregnant woman experiencing nausea and vomiting should: Eat small, frequent meals
(every 2 to 3 hours).
A pregnant woman reports that she is still playing tennis at 32 weeks of gestation. The Several glasses of fluid
nurse would be most concerned regarding what this woman consumes during and after
tennis matches. Which is the MOST important?
Women with an inadequate weight gain during pregnancy are at higher risk of giving birth Intrauterine growth
to an infant with: restriction
Which minerals and vitamins usually are recommended to supplement a pregnant Iron and folate
woman's diet?
With regard to nutritional needs during lactation, a maternity nurse should be aware that: Caffeine consumed by
the mother accumulates
in the infant, who
therefore may be
unusually active and
wakeful.
SAS 58
A laboring woman becomes anxious during the transition phase of the first stage of labor Help the woman breathe
and develops a rapid and deep respiratory pattern. She complains of feeling dizzy and into a paper bag.
light-headed. The nurse's immediate response would be to:
Woman is experiencing back labor and complains of constant, intense pain in her lower counter pressure against
back. An effective relief measure is to use: the sacrum
Nurses should be aware of the difference experience can make in labor pain, such as: sensory pain for
nulliparous women often
is greater than for
multiparous women
during early labor.
With regard to what might be called the tactile approaches to comfort management, hand and foot massage
nurses should be aware that: may be especially
relaxing in advanced
labor when a woman's
tolerance for touch is
limited.
It is based on the belief that people have control and can regulate internal events such as Biofeedback
heart rate and pain responses:
It the application of pressure or massage at these same points and it is the most effective Acupressure
for low back pain:
It is based on the concept that illness results from an imbalance of energy and to correct Acupuncture
the imbalance, needles are inserted into the skin at designated susceptible body points
(tsubos) located along meridians that course throughout the body to supply the organs of
the body with energy:
Mia Thermopolis is having labor and she was instructed by the Nurse-Midwife to stand Yoga and Meditation
under a warm shower or soaking in a tub of warm water, jet hydrotherapy tub, or
whirlpool is another way to apply heat to help reduce the pain of labor: You as a Student
Nurse knows that this is:
It offers a significant variety of proven health benefits, including increasing the efficiency Yoga and Meditation
of the heart, slowing the respiratory rate, improving fitness, lowering blood pressure,
promoting relaxation, reducing stress, and allaying anxiety
It is based on the concept that everyone’s body contains energy fields that, when Therapeutic Touch and
plentiful, lead to health or, when in low supply, result in illness: Massage
SAS 59
A 36-year-old woman tells the nurse that she has not had a menstrual period for 3 What was your
months and asks whether she is going into menopause. The best response by the nurse is menstrual pattern before
your periods stopped?
Which information will the nurse include when teaching a 31-year-old woman who is HRT decreases
considering the use of combined estrogen-progesterone hormone replacement therapy osteoporosis risk and
(HRT) during menopause? increases the risk for
cardiovascular disease
and breast cancer
The nurse provides drug teaching for a 30-year-old woman who is prescribed clomiphene I should avoid
(Clomid). It is most important for the nurse to follow up on which patient statement? intercourse while taking
this medication
Because of the risks associated with hormone therapy (HT), a 50-year-old patient does Decrease heat
not want to take HT and asks the nurse how she can handle her perimenopausal production and increase
symptoms of hot flashes and sweating at night. What should the nurse first advise this heat loss
patient?
Men and Women are equally likely to have fertility problems. True
These are the major factor for infertility in women. SELECT ALL THAT APPLY A. Age
B. Weight
C. Anovulation
D. Hormonal Therapy
Women who are trying to conceive should boost their intake of: Folic Acid
The most fertile days of a woman’s cycle can vary from month to month: True
Men do not experience age-related decrease in fertility. False
Which sexually transmitted disease can result in infertility in women? Pelvic Inflammatory
Disease
SAS 60
A group of nursing students at Elmira College is currently learning about family violence. Family violence affects
Which of the following is true about the topic mentioned? every socioeconomic
level
During a well-child checkup, a mother tells the Nurse Rio about a recent situation in which conflictual relationships
her child needed to be disciplined by her husband. The child was slapped in the face for of parents
not getting her husband breakfast on Saturday, despite being told on Thursday never to
prepare food for him. Nurse Rio analyzes the family system and concludes it is
dysfunctional. All of the following factors contribute to this dysfunction except:
During a home visit to a family of three: a mother, a father, and their child, The mother The nurse commends the
tells the community nurse that the father (who is not present) has hit the child on several mother's efforts and also
occasions when he was drinking. The mother further explains that she has talked her contacts protective
husband into going to Alcoholics Anonymous and asks the nurse not to interfere, so her services.
husband won't get angry and refuse treatment. Which of the following is the best
response of the nurse?
Nurse Ching is observing 6-year-old Anna during a community visit. Which of the The child has interest in
following findings would lead the nurse to suspect that Anna is a victim of sexual abuse? things of a sexual nature
Nurse Chika is working in the emergency department of Sky Castle Medical Center. She is Ensure Privacy
conducting an interview with a victim of spousal abuse. Which step should the nurse take
first?
Joseph, a 12-year-old child, complains to the school nurse about nausea and dizziness. Report suspicion of
While assessing the child, the nurse notices a black eye that looks like an injury. This is the abuse to the proper
third time in 1 month that the child has visited the nurse. Each time, the child provides authorities.
vague explanations for various injuries. Which of the following is the school nurse’s
priority intervention?
Carrie is studying about abuse for the upcoming exam. For her to fully instill the topic, she Implement measures to
should know that the priority nursing intervention for a child or elder victim of abuse is: ensure the victim’s
safety.
A community nurse conducts a primary prevention, home-visit assessment for a newborn Impaired parenting,
and mother. Mrs. Smith has three other children, the oldest of whom is age 12. She tells related to the role
the nurse that her 12-year-old daughter is expected to prepare family meals, to look after reversal of mother and
the young children, and to clean the house once a week. Which of the following is the child
most appropriate nursing diagnosis for this family situation?
Mrs. Chen’s was admitted to the emergency department of Wu Medical Center with a Mrs. Chen’s explanation
fractured arm. She explains to the nurse that her injury resulted when she provoked her is a typical response of a
drunken husband, Mr. Smith, who then pushed her. Which of the following best describes victim accepting blame
the nurse’s understanding of the wife’s explanation? for the abuser.
Cha tells the community nurse that her boyfriend has been abusive and she is afraid of A. Help Cha to develop a
him, but she doesn’t want to leave. The client asks the nurse for assistance. Which plan to ensure safety,
nursing interventions are appropriate in this situation? Select all that apply. including phone numbers
for emergency help.
C. Communicate
acceptance, avoiding any
implication that Cha is at
fault for not leaving.
D. Help Cha to explore
available options,
including shelters and
legal protection.
F. Reinforce concern for
Cha’s safety and her right
to be free of abuse.
FROM QUIZZES
On arrival at the emergency department, a client tells the nurse that she suspects that she ectopic pregnancy
may be pregnant but has been having a small amount of bleeding and has severe pain in
the lower abdomen. The client’s blood pressure is 70/50 mmHg and her pulse rate is 120
bpm. The nurse notifies the physician immediately because which of the following is
suspected?
A 36-year-old multigravid client is admitted to the hospital with possible ruptured ectopic episodes of pelvic
pregnancy. When obtaining the client’s history, which of the following would be most inflammatory disease
important to identify as a predisposing factor?
A multigravid client seen in the emergency department complaining of sharp abdominal fallopian tube
pain and vaginal spotting is diagnosed with an ectopic pregnancy. When explaining to the
client and family members about an ectopic pregnancy, which of the following would the
nurse include as the most common site of implantation?
After instruction of a primigravid client at 8 weeks’ gestation about measures to “I’ll eat two large meals
overcome early morning nausea and vomiting, which of the following client statements daily with frequent
indicates the need for additional teaching? protein snacks.”
When asked about the cause of H-mole, your answer will reflect an understanding that History of abortion
the exact cause of H-mole is: [unknown]
A 38-year-old client at about 14 weeks’ gestation is admitted to the hospital with a pregnancy-induced
diagnosis of complete hydatidiform mole. Soon after admission, the nurse would assess hypertension
the client for signs and symptoms of which of the following?
A multigravid client is admitted to the hospital with a diagnosis of ectopic pregnancy. The Methotrexate
nurse anticipates that, because the client’s fallopian tube has not yet ruptured, which of
the following may be ordered?
The factors that play a significant role in the causation of spontaneous abortion include: Infection, defective
ovum, diabetes mellitus,
incompetent cervix.
Nurse Kim, assessing Mrs. Joe, 33 weeks’ gestation with vaginal bleeding, is aware that an uterine tenderness upon
abruptio placenta is accompanied by which of the following assessment findings? palpation
A multigravida at 37 weeks’ gestation is admitted with painless, bright red bleeding and Cervical examination
mild contractions every 7 to 10 minutes. Which of the following assessments should be
AVOIDED?
A G4P3 woman is admitted to the prenatal clinic at 8 weeks’ gestation with a diagnosis of Assess the amount and
placenta previa, incomplete. Chief complaint is painless vaginal bleeding. Which action character of bleeding
should you perform INITIALLY?
A client with severe preeclampsia is admitted with a blood pressure of 160/110 mmHg, seizure precautions
proteinuria, and severe edema. Which of the following would be most important to
include in the client’s plan of care?
As part of the prenatal teaching, the nurse instructs the client to immediately report any forerunners to
blurring or dimness of vision accompanied by sudden weight gain. The BEST rationale for preeclampsia.
this instruction is that the symptoms are:
Prior to magnesium sulfate administration, what are the important parameters should a blood pressure,
nurse must assess? respiratory rate, and
body temperature
The nurse should inform the client with mastitis that the disorder is most commonly Staphylococcus aureus
caused by which of the following organisms?
Which of the following should the nurse do when a breastfeeding primipara tells the Tell her to breastfeed
nurse that she has sore nipples? more frequently.
What notable sign may indicate increased intracranial pressure in an infant with Overflow voiding
hydrocephalus?
[High-pitched cry]
After teaching the parents of a neonate born with a cleft lip and palate about appropriate “I should clean her
feeding techniques, the nurse determines that the mother needs further instruction when mouth with soapy water
the mother says which of the following? after feeding.”
After teaching the parents of an infant diagnosed with Hirschsprung’s disease, the nurse “The nerves to the end of
determines that the parents understand the diagnosis when the father states which of the large colon are
the following? missing.”
Which of the following conditions would the nurse expect when assessing a neonate for Depressed fontanel, eyes
hydrocephalus? rotated downward
Which of the following nursing diagnoses is MOST relevant in the first 12 hours of life for Risk for infection
a neonate born with a myelomeningocele?
Mr. and Ms. Byers’ child failed to pass meconium within the first 24 hours after birth; this Hirschsprung’s disease
may indicate which of the following?
The primary treatment of bacterial meningitis is Antibiotic
An infant returns from surgery after repair of a cleft palate. Which of the following Position the infant on his
nursing interventions should be done first? side
Which of the following factors can aid in neural tube defect detection? X-ray of the lower
abdomen after the 23rd
week of gestation.
Which of the following causative agents is the most common cause of bacterial Streptococcus
meningitis? pneumonia
Which of the following structural defects involves the use of Logan bow postoperatively? Cleft lip or palate

A child has just returned to the pediatric unit following ventriculoperitoneal shunt Place the child on the
placement for hydrocephalus. Which of the following interventions would the nurse side opposite the shunt.
perform FIRST?
Dr. Jones prescribes corticosteroids for a child with nephritic syndrome. What is the [To decrease
primary purpose of administering corticosteroids to this child? proteinuria]
A 6-year-old child is scheduled to have measles, mumps, and rubella (MMR) vaccine. Intramuscularly in the
Which of the following route will you expect the nurse to administer the vaccine? deltoid muscle

[Intramuscularly in the
vastus lateralis muscle.]
Which of the following would the nurse identify as the initial priority for a child with acute Instituting infection
lymphocytic leukemia? control precautions.
A teen patient is admitted to the hospital by his physician who suspects a diagnosis of A. Urine specific gravity
acute glomerulonephritis. Which of the following findings is consistent with this of 1.040
diagnosis? Select all that apply. B. Urine output of 350 ml
in 24 hours
C. Brown (“tea-colored”)
urine
A toddler diagnosed with nephrotic syndrome has a nursing diagnosis of Fluid Volume Weighing the child
Excess related to fluid accumulation in the tissues. Which of the following would the before breakfast
nurse anticipate including in the child’s plan of care?
A child is seen in the pediatrician’s office for complaints of bone and joint pain. Which of Petechiae
the following other assessment findings may suggest leukemia?
Nurse Kai is evaluating a female child with acute post-streptococcal glomerulonephritis Increased urine output
for signs of improvement. Which finding typically is the earliest sign of improvement?
A 6-year-old child with nephrotic syndrome has been hospitalized. She presents with a Daily measurement of
distended abdomen, periorbital edema, proteinuria, anorexia, and fatigue. Which nursing weight and abdominal
measure would be most appropriate? girth
Which of the following complications are three main consequences of leukemia? Anemia, infection, and
bleeding tendencies.
The client with varicella will most likely have an order for which category of medication? Antivirals

When educating parents regarding known antecedent infections in acute Impetigo


glomerulonephritis, which of the following should the nurse cover?
Which of the following conditions most commonly causes acute glomerulonephritis? Prior infection with group
A Streptococcus within
the past 10-14 days
The mother of a child with a ventriculoperitoneal shunt says her child has a temperature Consult the primary
of 38.4 degree celsius, blood pressure of 108/68 mmHg, and a pulse of 100. The child is health care provider.
lethargic and vomited the night before. Other children in the family have had similar
symptoms. Which nursing intervention is MOST appropriate?
After birth, early detection of hydrocephalus can be done by: observing the signs and
symptoms of
hydrocephalus
Which of the following would alert the nurse initially to suspect hydrocephalus in an Increased head
infant who has undergone surgical repair of a myelomeningocele? circumference and
bulging fontanel
Which of the following characteristics or conditions would the nurse expect to find in a Steatorrhea
child diagnosed with celiac disease?
An 11-year-old girl with celiac disease was discharged from the hospital. An appropriate Wheat
teaching was carried out by the nurse if the parents are aware of avoiding which of the
following?
Which of the following conditions would be considered a malabsorption disease of the GI Celiac disease
system?
While assessing a full term neonate, which of the following symptoms would cause the A weak sucking reflex
nurse to suspect a neurologic impairment?
Which of the following actions should be the nurse’s priority for an infant returning to the Place the infant on its
pediatric unit following repair of a myelomeningocele? abdomen.
A child with celiac disease is being discharged from the hospital. Which of the following Luncheon meat
food items would be included in his diet?
A 1-year-old child is brought to the emergency department with a mild respiratory Tugging on the ears
infection and a temperature of 3.5 degree celsius. Otitis media is diagnosed. Which of the
following sign is a characteristic of otitis media?
Children with chronic otitis media often require surgery for a myringotomy and ear tube To flush the middle ear
placement. Which of the following management strategies explains the purpose of the
ear tubes?
Which of the following statements about cerebral palsy would be accurate? Cerebral palsy means
there will be many
disabilities.

[Cerebral palsy is a
condition that doesn’t
get worse]
What is the most common therapy/strategy to help cerebral palsy? Physical therapy

Identifying factors that trigger seizure activity could lead to which of the following Avoiding striped
alterations in the child's environment or activities of daily living? wallpaper and ceiling
fans
A preschool-age child has just been admitted to the pediatric unit with a diagnosis of Decrease environmental
bacterial meningitis. The nurse would include which of the following recommendations in stimulation.
the nursing plan?
Which of the following durations would cause otitis media to be classified as chronic? Approximately 2 weeks

A school-age child with a diagnosis of epilepsy is admitted to the pediatric unit of a local Loosen any restrictive
hospital for evaluation of his anticonvulsant medications. as the nurse enters the child's clothing
room, the child begins to have seizure. Which nursing action should the nurse do first?
The nurse teaches a mother how to provide adequate nutrition for her toddler, who has The child eats finger
cerebral palsy. Which of the following observations indicates that teaching has been foods by himself
EFFECTIVE?
At which of the following times is seizure activity most likely to occur? During the rapid eye
movement (REM) stage
of sleep.

[While falling asleep and


on awakening.]
For which of the following symptoms will the nurse assess a neonate diagnosed with temperature instability,
bacterial meningitis? irritability, and poor
feeding

You might also like